Wikipedia:Reference desk/Humanities

This is an old revision of this page, as edited by 20.137.18.53 (talk) at 14:41, 3 October 2012 (→‎Which part of the paperclip product chain is, on average, the most profitable?: new section). The present address (URL) is a permanent link to this revision, which may differ significantly from the current revision.


Welcome to the humanities section
of the Wikipedia reference desk.
Select a section:
Want a faster answer?

Main page: Help searching Wikipedia

   

How can I get my question answered?

  • Select the section of the desk that best fits the general topic of your question (see the navigation column to the right).
  • Post your question to only one section, providing a short header that gives the topic of your question.
  • Type '~~~~' (that is, four tilde characters) at the end – this signs and dates your contribution so we know who wrote what and when.
  • Don't post personal contact information – it will be removed. Any answers will be provided here.
  • Please be as specific as possible, and include all relevant context – the usefulness of answers may depend on the context.
  • Note:
    • We don't answer (and may remove) questions that require medical diagnosis or legal advice.
    • We don't answer requests for opinions, predictions or debate.
    • We don't do your homework for you, though we'll help you past the stuck point.
    • We don't conduct original research or provide a free source of ideas, but we'll help you find information you need.



How do I answer a question?

Main page: Wikipedia:Reference desk/Guidelines

  • The best answers address the question directly, and back up facts with wikilinks and links to sources. Do not edit others' comments and do not give any medical or legal advice.
See also:


September 28

Math

What is the difference between USAMO and USAJMO beside the obvious fact that USAJMO is for 10 grade or below and USAMO is for anyone who capable of doing it. The USAMO and USAJMO I'm talking are the American Mathematic Contests. I know the top 6 smartest scorer in the USAMO will get to represent the USA in International Mathematical Olympiad. So what do top 6 in USAJMO get? What is the point of doing USAJMO? Just for the sake of practice for USAMO or something?65.128.190.136 (talk) 04:06, 28 September 2012 (UTC)Reply

For those strange people who need spoon-feeding, USAMO is United States of America Mathematical Olympiad. -- Jack of Oz [Talk] 05:55, 28 September 2012 (UTC)Reply

What's the point of winning the World Junior Chess Championship? It's not like it gives you the right to challenge for the World Chess Championships? Just for the sake of practice for the World Champs or something?

I'll answer your question and mine: see our articles on prestige and (loosely) sport. --Dweller (talk) 11:45, 28 September 2012 (UTC)Reply

Emergency dispatchers

So twice now I have called a local police department and both times the dispatcher has seemed to get off the phone as quickly as possible. The first time I called 911 (UK's 112 999, Canada's 112) to report a funnel cloud and the second time the non-emergency line to report possible drug activity. The first time basically the moment I told the dispatcher about the funnel cloud I got a response of "We are aware of it and have people monitoring the situation" and a prompt end of the call. The second time I reported my suspicions and the dispatcher said they would have someone check it out then promptly said "Alright, I have to go" and hung up. I was under the impression that generally the dispatchers would want to be more thorough in collecting details than that (with the first case asking if I was a spotter, etc, and in the second collecting information about the people involved), so why would they rush to get off the line? Ks0stm (TCGE) 06:50, 28 September 2012 (UTC)Reply

You mean 911 is the US's 112 not the UK's which is 999. You will have to ask the local authorities or local politicians who set the priorities for the service. Here in UK we get the same complaints and one reason given is the increase in 'emergency' calls being made as a result of people carrying mobile phones. Sussexonian (talk) 07:06, 28 September 2012 (UTC)Reply
Whoops, yeah, my bad. I don't know why I remembered 112 for the UK, it's Canada's. 999 for the UK. Ks0stm (TCGE) 07:22, 28 September 2012 (UTC)Reply
112 is European, of course, and is an alternative to 999 in the UK. In my only experience of using the emergency number, the operator remained on the line much longer than I expected, in fact, until the police arrived. I think it all depends on the content of your call and an estimate of the danger someone might be in. Dbfirs 07:43, 28 September 2012 (UTC)Antwort
I'm Canadian, and this is the first I've ever heard of "112". The 112 (emergency telephone number) article even claims that 112 redirects to 911, so i would hardly call 112 a Canadian number. We are decidedly a 911 country. Mingmingla (talk) 18:59, 28 September 2012 (UTC)Reply
It sounds like they were overloaded with calls, so, as soon as they determined that nobody's life was in danger, they went on to the next call. StuRat (talk) 08:10, 28 September 2012 (UTC)Reply
Sussexonion & StuRat are right IMO. There seem to be quite a lot of negligence calls, usually because the 9 button of the phone is jammed against some object while the phone is in a pocket or backpack (so the owner doesn't even notice). Holding 9 down for 2 seconds will cause a 911 on many mobiles. On the local radio I've heard that these calls are more frequent than real emergency calls. - ¡Ouch! (hurt me / more pain) 08:21, 28 September 2012 (UTC)Reply
OTOH, it might simply be the case that the funnel cloud may have already been reported to the very same operator, possibly by multiple callers. Same possibly with the drug users. Just a guess though. It might be as well that you actually beat others to the call, and the operator saw his call queue fill up, so he hurried through your report in case there was a more severe emergency among the queued callers, - ¡Ouch! (hurt me / more pain) 08:21, 28 September 2012 (UTC)Reply
Some refs back up the above re the need to work quickly and the decision-making process: This article gives some interesting statistics on the volume and nature of 911 calls in a small U.S. county (pop. 21062). Dispatchers field 911, numerous other calls. See also What does a 911 dispatcher do. Taknaran (talk) 15:07, 28 September 2012 (UTC)Reply
Can you clarify why you were calling 911 about "possible drug activity"? Was someone being forced to buy drugs against his will? What made you suspect immanent physical danger? μηδείς (talk) 20:31, 28 September 2012 (UTC)Reply
I believe you can call 911 when a felony is being committed, even a victimless one. StuRat (talk) 23:11, 28 September 2012 (UTC)Antwort
Like embezzlement? Yeah.... My understanding of the emergency responder line is that it is for emergency responses. Last time (and only) I called 911 was when I heard my neighbor's wife shoot him. μηδείς (talk) 23:17, 28 September 2012 (UTC)Antwort
Did you mean immanent before? -- Jack of Oz [Talk] 02:34, 29 September 2012 (UTC)Antwort
Ahh, no, for the funnel cloud (imminent threat to life/property from the weather if it were to touch down) I called 911, but for the suspected drug activity (not much urgent, just a group of people which appeared to be dealing/using drugs in a parking lot right my my university's residence halls) I called the non-emergency line since there was not imminent threat of damage or injury. Ks0stm (TCGE) 09:50, 29 September 2012 (UTC)Reply
Yeah, 911 is supposed to be for emergencies only. Call your local police department or sheriff's office to report possible illegal activity. — The Hand That Feeds You:Bite 02:21, 29 September 2012 (UTC)Antwort
In case there remains any confusion, the OP clealy specified from the first post ([1] in case there's any suggestion it was modified), they called the non emergency number to report the possible drug activity: "the second time the non-emergency line to report possible drug activity". Nil Einne (talk) 18:55, 29 September 2012 (UTC)Antwort
My bad, I missed the non-, sorry for the confusion. μηδείς (talk) 19:28, 29 September 2012 (UTC)Reply
BTW, when my neighbor was shot, and I called 911, I said "I'm at XXX MY Avenue, fourth floor, someone's been shot" they said "it's already been reported, stay in your apartment" and hung up, five seconds or less. The police were there within 30 seconds. The response was breathtaking, impressive, and though I have been very critical of the NYPD, very reassuring. μηδείς (talk) 19:37, 29 September 2012 (UTC)Reply
I've called 911 twice to report medical emergencies (never a police emergency) and I never was left with the impression that they were trying to get me off the line quickly. I was the first reporter in both cases. I suspect in your case, and the case just above, that it's about whether they already know about something and don't want to clog up the line redundantly. --Mr.98 (talk) 02:14, 30 September 2012 (UTC)Antwort
Yes, I think it was both obvious nothing more needed to be communicated and they probably still had calls coming in since there were around 100 apartments in that building. μηδείς (talk) 17:01, 30 September 2012 (UTC)Reply

Property Rights Question

I have a question--if hypothetically, say, someone had an expensive autographed baseball that was his property which he accidentally threw past his neighbor's fence, and the neighbor refuses to give the autographed baseball back, does the owner of the baseball have a right to demand and even take legal action to get his baseball/property back? Has there ever been any similar scenario to this and is there any knowledge or speculation about how courts might rule in such a situation? Futurist110 (talk) 06:53, 28 September 2012 (UTC)Reply

OK. Doubtless this will turn into a squabble debate about legal advice, despite the use of "hypothetically", but a non-advisory answer is possible. And that answer, as usual, is it depends on the jurisdiction. As an example of a similar scenario, under the law of England and Wales, deliberately keeping someone else's property that has accidentally ended up on your land when the undisputed owner has requested its return is theft, and here is a newspaper article showing how the police handle such situations when a complaint is received. For other jurisdictions you would have to check what the applicable law says. As for speculation about how the courts might rule, this would be inappropriate on the RefDesk and not a whole lot of practical use in the real world, either. I'm hoping we can, between us, demonstrate how to handle this question per policy without provoking a removal or a hatting, but I'm not holding my breath. - Karenjc 08:43, 28 September 2012 (UTC)Reply
Yup... it depends on the Jurisdiction... consider the situation where someone's cow wanders off and ends up in someone else's field. Who owns the cow? The answer has been different in different places (and even in the same place at different times in history). Blueboar (talk) 14:19, 28 September 2012 (UTC)Antwort
The issue of a cow on someone else's field usually arises because one can identify the field, but not the cow. That's not the case if the cattle are branded. This is not some generic ball. I would love to see a reference to a legal case where a uniquely identifiable item was deemed someone else's property because it was inadvertently misplced on someone else's land. μηδείς (talk) 20:28, 28 September 2012 (UTC)Antwort
Detinue, Replevin, Trover and Conversion (law) are all relevant to the question to an extent, and make interesting reading for the historical perspective. - Karenjc 21:36, 28 September 2012 (UTC)Reply

Bart Simpson's grossest foods

Hello L.H. ! . I am trying to add on the § « Références culturelles » of the french version of The Food Wife , & I see that among the grossest foods Bart boasts in his blogs of having tasted are : "leech cheeks, bear oysters, charlie’s trotters, duck butter, krustyburgers" . For "bear oysters", I discard Acanthus mollis, & rather think of testicle (food) , since they are a lot of bears in the Rockies. For "charlie’s trotters", I think of Viet Cong hams (certainly not too fat…) …. For ""duck butter"", I can’t find anything on WP, but I remember in june 2009 I read somewhere that a widely known just-deceased star had the expression « duck butter » ready as an explanation for the young ones he was so fond of when they questionned him. But I can’t find the reference. Can you help me ? . Thanks a lot beforehand. T.Y; Arapaima (talk) 10:42, 28 September 2012 (UTC)Reply

They're so surreal you might as well go for the direct equivalents. Joues de sangsue, huitres d'ours, pieds de Charlot, beurre de canard, krustyburgers. Makes absolutely no sense at all, but then it doesn't in English. The hunches about oysters and Charlie Trotter could well be right, but you can't capture everything in translation. Itsmejudith (talk) 13:28, 28 September 2012 (UTC)Reply
Arapaima, have these episodes aired in French yet? You could just use the translated terms. There are transcriptions of most of the French episodes at SimpsonsPark (although not this episode, not yet). Adam Bishop (talk) 16:43, 28 September 2012 (UTC)Reply
You'll find assorted definitions of "duck butter" by entering those words in double quotes plus "urban dictionary" in your search engine of choice. It's something particularly vulgar and might be a regional term rather than "urban" per se. Remember that "Bart Simpson" is a cartoon character in a series written by - and, arguably, for - adults. There's a long tradition among scriptwriters and their ilk of slipping stuff past the editors and/or over the heads of the intended audience. -- Deborahjay (talk) 14:01, 29 September 2012 (UTC)Reply
Thanks awfully to all ! Of course, as we say here, "tout est pur aux purs" ("everything looks pure to the pure ones") , but I maintain that, if you really look for it (slow motion, image freezing) , you’ll find The Simpsons are a most unexpected outlook on US culture. Especially if you read the titles of the books on the shelves, the shop posters, scrutinize the pictures hanging from the walls and the "things to do" lists etc… Listen, for exemple, in Wacking Day, to Marge asking Homer, in her hoarse voice, how he is going to use this new bludgeon he is taking out of its sheath. And in The Food Wife , we see Homer about to sniff meth from a properly heaten glass balloon in a lab the Spuckler family ( parents and their whole dozen of children) patrons… T.y.Arapaima (talk) 05:50, 1 October 2012 (UTC)Reply

Handly's Lessee v. Anthony

Where is the peninsula in question in Handly's Lessee v. Anthony? From the description given in the article, I can't figure it out, because there are many places that fit the general description, and I don't myself know about any meanders that have streams as described here. I've tried Google but found nothing. Anyway, a modern map might not help, because dams built for the Ohio River canalization process have raised the water levels in many places and changed the shorelines accordingly. 2001:18E8:2:1020:60FB:F516:1E7C:B1D3 (talk) 16:04, 28 September 2012 (UTC)Reply

Green River Island.—eric 14:56, 29 September 2012 (UTC)Reply

United States electoral college system

As a citizen of the United Kingdom I don't claim to know much about the American electoral system. What I do understand is that states are granted electoral college votes based on their population, and that when a democrat or repubican candidate gets a majority all of the electoral college votes are awarded to that candidate. What I dont understand is in states like California, New York and Florida, where there are a lot of electoral college votes to be obtained, why are the votes not evenly distributed between the two nominees. For instance, lets assume theres 50 million people in California; if 26 million people voted democrat and then 24 million voted republican, why would the democrat candidate get all of the 55 available votes? Surely it should be averaged out between the two candidates. Doesn't make much sense to me but it doesn't seem particularly democratic. --Toryroxy (talk) 17:05, 28 September 2012 (UTC)Reply

You can read more about it at "Electoral College (United States)", specifically the "Criticism" and "Support" sections. Gabbe (talk) 17:29, 28 September 2012 (UTC)Reply
With the exception of Maine and Nebraska, you are perfectly correct. It does not make much sense and it is far from democratic. Many Americans realize how antiqued and broken the system is and are working towards fixing it. The National Popular Vote Interstate Compact will effectively abolish the electoral college and it's already 49% to its goal despite being only 5 years old. A8875 (talk) 17:33, 28 September 2012 (UTC)Antwort
Partisan answers aside, the reason that it works that way is that the Constitution permits each state to decide how its electoral votes are cast, and most state legislatures have chosen to use a winner-take-all format. Note that all interstate compacts require the approval of Congress in order to become law. Nyttend (talk) 17:55, 28 September 2012 (UTC)Reply
This American is disgusted that the NPVIC would hand a landslide to a plurality winner, not only to a majority winner. —Tamfang (talk) 05:19, 29 September 2012 (UTC)Reply
I believe the standard argument is that splitting the seats like that would reduce the influence of the state, since most of their seats would just cancel each other out (looking at the 2008 results, apart from DC and a few states with only 3 or 4 seats, the largest majority was 65%, which means only 30% of that state's seats would actually count for anything, and it's far less for most states). The only way to implement a more democratic system without weakening your own position is if everyone else (or, at least, a very large proportion) goes along with it too, which is the point of the interstate pact mentioned above. --Tango (talk) 18:31, 28 September 2012 (UTC)Reply
The system may not be perfect, but it was a lot more democratic than having a King and a Parliament dominated by the House of Lords determine things... which was how things ran back in England before the Americans declared their Independence. It is also important to remember that, in 1787 (when this was all set up), each State was essentially an independent nation... and they did not fully trust each other. New York, for example, was not about to agree to something that might give more political clout to Virginia. So each State insisted that it set its own rules on how to choose Electors. And it was in each State's interest to have all its Electors form a solid block (all voting for same Presidential candidate), rather than a a divided slate that was split proportionally between more than one candidate. Thus, the preference for "Winner-take-all" in most states. Blueboar (talk) 18:33, 28 September 2012 (UTC)Reply
  • Surely now though its time to rethink that as it happened two and a bit centuries ago and now the United States is a more stable political entity. Also the explanation you provided seems to refer mostly to historic states choosing the winner takes all system, I don't see why the western states would adopt this policy. I think it would be better if in the US election you forgot about all the states and the nation voted as a whole. — Preceding unsigned comment added by Toryroxy (talkcontribs) 19:19, 28 September 2012 (UTC)Reply
Except, you came here to ask why it is the way it is - which we have answered. Not to argue for why it should be changed - which would be a debate, which we cannot accommodate. -- Jack of Oz [Talk] 19:52, 28 September 2012 (UTC)Antwort
I have to agree 100% with Nyttend, Tango, Blueboar, and 141.59% with JackofOz, whose pretty yellow signatures I miss. μηδείς (talk) 20:19, 28 September 2012 (UTC)Antwort
It ain't happening. You'll survive. -- Jack of Oz [Talk] 22:05, 28 September 2012 (UTC) Antwort
The notes are missing. Has the music died? :(
OK, every time a U.S. Presidential election comes around, a few folks here and there will question the somewhat bizarre setup we call the Electoral College. It may be "undemocratic" in some sense, but there's no serious move to change it. It would require a constitutional amendment, and it is extremely unlikely any of the "red states" (Republican) would go for it, so, to coin a phrase, "It ain't happening." ←Baseball Bugs What's up, Doc? carrots05:09, 29 September 2012 (UTC)Antwort
The "compact" thing actually has a chance of working. I'm almost sure it's constitutional, without any need for an amendment. I'm almost equally sure it's not enforceable without an amendment — if some state decides to back out after the election, things could get entertaining. --Trovatore (talk) 05:13, 29 September 2012 (UTC)Antwort
The issue with the Electoral College is that it certainly isn't any less democratic than how other recognized democratic governments elect their leaders. In the UK, the voters have no direct control over who gets to be in The Government. They elect their MPs, who in tern put forward a Prime Minister nominee who the Monarch rubber stamps. That Prime Minister then selects everyone else in The Government, who the Queen then rubber stamps. The British Electorate has no more say in who gets to be Prime Minister than the U.S. electorate does in who gets to be President. In many ways, the U.S. system is more direct, as people at least choose the slate of electors who is pledged to vote for the President. It's a step removed from selecting the President directly, but not nearly as removed as the British system is. At the British system still allows people to directly elect their legislator who represents their single-member district. Other western democracies use proportional representation systems, whereby people vote for the parties primarily, and have little say over which candidate directly represents them. It doesn't make one system or another better, or one system or another more or less democratic. They all have their benefits and their faults, different does not equal worse. --Jayron32 05:23, 29 September 2012 (UTC)Antwort
I wasn't arguing against the Electoral College. I was just saying that the compact idea is probably constitutional (at least, with the consent of Congress, but that's easier to get than an amendment), but probably not enforceable. --Trovatore (talk) 05:31, 29 September 2012 (UTC)Antwort
Sorry, I indented one too many. I hadn't meant that as a response to you. My answer was meant to be a response to Bugs's assertion that the Electoral College (because it isn't "direct democracy") was, in his words "may be "undemocratic" in some sense". I was just trying to correct that misconception by showing it as being as democratic as any other system used to elect leaders. --Jayron32 06:49, 29 September 2012 (UTC)Antwort
It's "undemocratic" in the sense that we don't vote directly for the President. But it's the system the founding fathers devised, that states elect the President, and there's no serious move to change it. With a very short list of exceptions, the President has usually been the one the majority (or at least the plurality) voted for anyway. Although it would be interesting to see which party would gain the advantage if the electoral votes were apportioned in every state instead of being winner-take-all in most states. I suspect the results would be worse than the current system, i.e. that the popular vote minority would be more likely to win consistently. And that would certainly not be "democratic". ←Baseball Bugs What's up, Doc? carrots19:08, 29 September 2012 (UTC)Reply

In the Federalist No. 39 (text here) James Madison describes at length how certain parts of the government under the Constitution have a "national" character and others a "federal" character. Congress, he says, has both—the lower House is "national", that is it represents the people rather than the states, while the upper Senate is "federal", with each state having equal power (2 senators each) regardless of population. The election of the president, he argues, also has mixed national and federal aspects. Although he doesn't explicitly mention the Electoral College in this essay it is clear that that is what he is talking about with regard to the "federal" aspect of the election of the president. Throughout this essay he tries to explain why it is important for the government to have a mixed national/federal character. It is interesting to contrast with Federalist No. 68 ([2]), in which Alexander Hamilton explains the Electoral College as essentially a buffer from the "heats and ferments" of the people at large. This argument of Hamilton's would not convince many people today. Madison's thoughts, on the other hand, I have always found interesting and fairly convincing. Pfly (talk) 22:27, 28 September 2012 (UTC)Reply

It also may be worth pointing out that, originally, electors were not intended to be rubber stamps, committed to a particular candidate. They were to be chosen by the state legislatures (who in turn were chosen by the people), and then to vote as they thought best. Hamilton's idea, whether you agree with it or not, is at least coherent in that context.
Today, electors' individual judgment counts for (almost) nothing, legislatures do not choose electors (though they still could if they wanted to), and so what we have is not so much indirect election as it is a form of direct popular election, just with a complicated way of deciding who won. In that context, it's hard to see how it provides any significant buffer against popular ferment. --Trovatore (talk) 22:51, 28 September 2012 (UTC)Antwort
Yes, Hamilton's argument was certainly coherent at the time, and yes, things have changed since he wrote that essay. Also at the time there was some fear that too much democracy would lead to mob rule and tyranny of the majority. No one knew how much "buffer" was needed (if any), nor how "dangerous" democracy would turn out to be. Hamilton is known for having been concerned about this, but Madison (and others) were too. An example I found interesting was how Madison advocated that new states be created with little regard for natural or social terrain. Unlike Jefferson who wanted new states to be small and more or less culturally cohesive, Madison wanted new states to be "large and rectangular" and take in disperate social and cultural patterns if possible—arguing that forcing differing interests/factions to work together helped avoid the tyranny of the majority—a "checks and balances" kind of thing. Looking at a map of the US today it is obvious that Madison's idea was the one adopted.
In any case, yes, things have changed and Hamilton's writing about the Electoral College doesn't apply so well to the system today. Madison's writing still does, I think, even though things have changed. When he wrote of the Senate being "federal" rather than "national" he was not only thinking of there being equal representation among the states in the Senate, but also that, at the time, state legislatures, not the people, elected or appointed senators. That, and the changed nature of the Electoral College, undermines his arguments somewhat, but the general logic of a hybrid national/federal system still holds. Other than in Maine and Nebraska all electors in a state vote for the same candidate even if that candidate only won by a small margin. So the president is still chosen through a mixed national/federal system. I'm not personally arguing that this is for the best, just that it seems to be what Madison was arguing for: The "national" (the people but also the mob) checked and balanced against the "federal" (the states). Pfly (talk) 23:43, 28 September 2012 (UTC)Antwort
Also, please remember that the electoral votes are not automatically cast. There are actual people voting in the Electoral College. They are almost always loyal party members, who are certain to vote for their party's candidate. However, once in a while one of these electors bucks the system and votes for someone else. See the article on "faithless electors".    → Michael J    02:46, 29 September 2012 (UTC)Reply
One very basic consideration was that given the geographical scope of the U.S. and the state of transportation and communication ca. 1787, some were worried that people in one state simply wouldn't know enough about politicians from other states to have a truly informed opinion. As late as the United States presidential election of 1824, there were several regional candidates for president, but no real national candidate. Also, each state set its own voting qualifications. It would have required much more federal-government-imposed standardization and uniformity than most people would have been willing to accept at that time in order to make it feasible to lump votes from different states into one big vote pool... AnonMoos (talk) 03:42, 29 September 2012 (UTC)Reply
An interesting reason for indirect elections is hidden in the wording of the (original) procedure of the Electoral College: "The electors shall ... vote by ballot for two Persons, of whom one at least shall not be an Inhabitant of the same State with themselves." This provision, intended to dilute local preferences in favor of candidates with wider appeal, would be harder to enforce with direct elections. (I suspect this rule is why the office of Vice President exists at all!) —Tamfang (talk) 05:19, 29 September 2012 (UTC)Antwort
The purpose for having a Vice President is so that there is someone theoretically able to take over the job immediately if something happens to the President. As regards the different states, it would be easy to enforce: If a Presidential candidate selects a VP candidate from the same state, he's not allowed to be elected, no matter how the popular vote turns out. ←Baseball Bugs What's up, Doc? carrots19:12, 29 September 2012 (UTC)Antwort
Even when there's no Vice President, there is a list of officers designated to take over the Presidency. Note by the way that even with Amendment XII the Constitution does not formally recognize that candidates for President and Vice-President run jointly. —Tamfang (talk) 21:37, 29 September 2012 (UTC)Antwort
Yeah, I thought it would have been fun in 2000 if Dick Cheney's slightly iffy claim to have re-established Wyoming residency had been rejected by the courts, meaning that Texas's electors could not have voted for both Bush and Cheney. If they had stuck with Bush (he needed them all, I think, or at least almost all), and picked someone else for veep, then the VP election would have been decided in the Senate, which I think was Democratic at the time. Can you say Bush–Lieberman administration? I knew you could. --Trovatore (talk) 21:44, 29 September 2012 (UTC)Reply

Intrinsic vs. Extrinsic religiosity

Can anyone explain to me the difference between intrinsic and extrinsic religiosity? What does it mean 'to be religious to an end'? Psychologically speaking, why is intrinsic religiosity better than extrinsic religiosity or irreligiosity? How does a person become religious? Are some people just born that way? What does that say about apostates (people who leave the faith) and religious converts (people who enter the faith)? My former psychology textbook talked something about intrinsic religiosity vs. extrinsic religiosity. My impression was that being intrinsically religious was better than extrinsically religious or religious to an end. No idea what that meant. If an atheist who embraces secular humanism disguises himself or herself as, say, a Christian and attends a traditional Christian church, then would that count as extrinsic religiosity? What happens if that atheist uses Jesus's teachings to the philosophy of religious humanism? Meanwhile, the Christian who is so-called "born-again" is intrinsically religious, even though he may be explicitly hateful and bigoted and arrogant and selfish and greedy? That does not make sense. 140.254.226.242 (talk) 20:23, 28 September 2012 (UTC)Reply

I'd guess that the answer to your initial question is: how heartfelt is it? Are you convinced of the religious principles that you hold, or are you just claiming to hold them in hopes that it will further your nonreligious goals? Imagine that you're an atheist in Saudi Arabia — you're still going to be extrinsically religious (i.e. you go through the rituals, even though you reject them at heart), because you'll face severe penalties for abandoning Islam. You're "religious to an end" here, because your end is survival. Note that this use of "end" does not mean "finish"; this is a sense for "end" in which the OED's definition of "An intended result of an action; an aim, purpose" applies. Nyttend (talk) 01:48, 29 September 2012 (UTC)Antwort
Are you using these terms intrinsic and extrinsic religiosity on your own, or is this coming from some source or article? I'd like to see how the terms are being used. μηδείς (talk) 19:14, 29 September 2012 (UTC)Reply

British royals and languages

Has any British royal ever master or attempt to master all the languages of the British Isles? That is Scottish Gaelic, Welsh, and English (I skipped Lowland Scottish because it is just English with an accent in my own opinion). The Tudors spoke some Welsh but not Gaelic; has any monarch since Elizabeth I of England learn or attempt to learn this language (this part is restricted to actual reigning monarchs sense I now know Prince Charles speaks Welsh)? James IV of Scotland was the last Scottish king to speak Gaelic; has any royal since learn or attempt to learn this language? Also has any British monarch learn or attempt to learn Irish?--The Emperor's New Spy (talk) 20:30, 28 September 2012 (UTC)Reply

According to The Old Man of Lochnagar "He (Prince Charles) has also read it in Welsh and Scottish Gaelic translations on television; he is not fully fluent in either language." He does take an active interest in Scottish Gaelic according to BBC: Prince Charles comments on row over Gaelic in Caithness. You forgot to mention Cornish, Manx Gaelic, Irish Gaelic, Ulster Scots, Guernésiais, Jèrriais and Sercquiais for traditional languages, and British Sign Language. I would imagine that these minority languages are far exceeded numerically by those who speak Urdu and other languages of immigrant communities. Alansplodge (talk) 21:45, 28 September 2012 (UTC)Reply
Not really an answer to your question, but Karen Blixen reported Edward VIII speaking Swahili to some Kikuyu Chiefs (in Shadows on the Grasss). Our article doesn't seem to mention much about his linguistic ability, other than learning a few words of Welsh. Zoonoses (talk) 02:47, 29 September 2012 (UTC)Reply
As an Englishman who lived in Lowland Scotland for several years (and who thus unconsciously acquired a Scottish accent, though my use of Scots language idioms when in Scotland was partially deliberate) I have to take issue with your opinion, Spy, that "Lowland Scottish . . . is just English with an accent."
Scots is, in my extended experience, a similar but nontheless distinct language from English – the two are sisters that diverged from their common roots many centuries ago, though in the modern era Scots is becoming more Anglicised. To be sure most Scots can, and when speaking to – or in the presence of – a non-Scot usually do, speak Scots-accented English, perhaps larded with a few Scots idioms. When not consciously or unconsciously doing so (as a form of register-adjustment) however, their vocabulary and grammar differs from standard English a lot more than one might realise if one has not oneself become, as I did, an "apparent Scot", to the extent that a listener with only native or acquired standard English would be unable to follow much of it. {The poster formerly known as 87.81.230.195} 84.21.143.150 (talk) 10:03, 29 September 2012 (UTC)Antwort
Scots has indeed acquired the kudos of a "separate language", though, in reality, there is a continuum of dialects of northern English from the Midlands to the Lallands. Scots shares most of its vocabulary with neighbouring dialects of English, some of which are closer to Scots than to standard English. This is not surprising, because they all developed from Northern Middle English. Dbfirs 15:59, 29 September 2012 (UTC)Reply
I meant fluent not knowing a few words.--The Emperor's New Spy (talk) 18:19, 1 October 2012 (UTC)Antwort
A fairly rigorous Google search suggests that the Prince of Wales is the most linguistically capable member of the House of Windsor in terms of Celtic languages, and that in a rather limited way. Note that appreciation of the merits of Celtic languages by Anglophones is really a 20th century phenomenon, if you exclude enthusiasts like George Borrow. Whether the Duke and Duchess of Cambridge are using their time in Ynys Môn to acquire a smattering of Welsh, I don't know, Alansplodge (talk) 12:47, 2 October 2012 (UTC)Antwort
I seem to have missed Prince Williams learns Welsh and Duchess Kate Learning Welsh. Another report says Prince William learns Swahili, a useful language in some parts of London. Alansplodge (talk) 12:53, 2 October 2012 (UTC)Reply


September 29

Mizrahi and sephardi jews neighbourhoods in Israel

Which cities of Israel has neighbourhoods where there are significant numbers of Mizrahi and Sephardi Jews regardless they are Haredi, Hasidic, Modern Orthodox, Reform/Progressive and Masorti/Conservative Jews? — Preceding unsigned comment added by 65.92.151.92 (talk) 01:04, 29 September 2012 (UTC) --65.92.151.92 (talk) 01:11, 29 September 2012 (UTC)Don MustafaReply

I would assume that there would be more Mizrahi Jews in Israel cities which are poorer and more conservative, such as Jerusalem. Mizrahi Jews are on average less educated and more conservative than Ashkenazi Jews. Futurist110 (talk) 02:30, 29 September 2012 (UTC)Reply
It appears that I was wrong. Based on table 2.23 here (http://www1.cbs.gov.il/reader/), the districts in Israel where a majority of Jews (excluding those whose fathers were born in Israel) were originally of Asian or African origin are the Southern District, but Jews of Asian and African origin (Mizrahi Jews) are also close to a majority (45.00+%) (excluding those whose fathers were born in Israel) in the Northern and Central Districts of Israel. Futurist110 (talk) 02:47, 29 September 2012 (UTC)Reply
I suggest evaluating cities by age (i.e. how many generations have passed since founding), and consider that flourishing locales with employment opportunities and available housing would attract internal migration of all demographic groups, while stagnating locales would suffer exmigration (not necessarily to elsewhere in Israel, but abroad). More recent immigration would have been directed towards underdeveloped, underpopulated locales, particularly the "development towns." Studying patterns (often called "waves") of immigration is highly relevant. Note that small or communal settlements such as moshavim and kibbutzim founded by ideological groups are more likely to be and remain demographically homogeneous, particularly in requiring membership or acceptance by a screening committee. Also, some later immigrant groups don't fit the conventional definitions of Sephardic or Mizrahi but parallel and compete with them for socioeconomic niches, including housing. -- Deborahjay (talk) 12:17, 29 September 2012 (UTC)Reply

In regards to the table that I linked to, keep in mind that it might not be completely accurate since more Mizrahi Jews have fathers born in Israel than Ashkenazi Jews (most Asian/African migration into Israel occurred before 1975, while there were hundreds of thousands of Jewish immigrants from the USSR and former USSR after 1975). Futurist110 (talk) 00:16, 30 September 2012 (UTC)Reply

candidates Israel election Shas and Meretz Parties

Is there a website where I can find the candidates of Shas and candidates of Meretz for 2009 Knesset elections? — Preceding unsigned comment added by 65.92.151.92 (talk) 01:09, 29 September 2012 (UTC) --65.92.151.92 (talk) 01:11, 29 September 2012 (UTC)Don MustafaReply

You mean the whole candidate list or just the ones that successfully managed to enter the Knesset? Futurist110 (talk) 01:36, 29 September 2012 (UTC)Antwort
The page List of members of the eighteenth Knesset lists the members by faction, evidently in the order they appeared on the slate. For those in lower positions (if that's what the OP requires), check the individual faction's official website appearing under External links on its Wikipedia page here (or in Hebrew if possible), using its Contact Us feature if you don't find information on the site. You might otherwise try sending a query to the Knesset's English-language website, or check the archives of Haaretz in English by dates. -- Deborahjay (talk) 11:55, 29 September 2012 (UTC)Antwort
Yes, the whole candidate list of Shas and Meretz.--65.92.152.148 (talk) 01:43, 30 September 2012 (UTC)Don MustafaReply

Honestly, guys, I couldn't find the list of the candidates of Shas party and Meretz party for the 2009 Knesset elections. Please help me. I just want to know.

candidates each political parties last elections Sweden Norway Denmark Belgium Netherlands Italy Spain Portugal

Is there a website that shows the candidates of each party (e.g. Social Democrats, Moderate, Sweden Democrats etc) in Sweden's last election and the name of the seats they were contesting in? Is there a website that shows the candidates of each party (e.g. Labour, Conservative, Progressive etc) in Norway's last election and the name of the seats they were contesting in? Is there a website that shows the candidates of each party (e.g. Social Democrats, Danish People's, Liberal etc) in Denmark's last election and the name of the seats they were contesting in? Is there a website that shows the candidates of each party (e.g. Labour, PVV, VVD etc) in Netherlands' last election and the name of the seats they were contesting in? Is there a website that shows the candidates of each party (e.g. Socialist, New Flemish Alliance etc) in Belgium's last election and the name of the seats they were contesting in? Is there a website that shows the candidates of each party (e.g. the people of Freedom, Democratic Party etc) in Italy's last election and the name of the seats they were contesting in? Is there a website that shows the candidates of each party (e.g. socialist workers party and peoples party) in Spain's last election and the name of the seats they were contesting in? Is there a website that shows the candidates of each party (e.g. Socialist and social democratic) in Portugal's last election and the name of the seats they were contesting in? This is not a homework question. I am just curious about who were the candidates of the political parties and what were the name of the seats. Thank you.--65.92.151.92 (talk) 01:31, 29 September 2012 (UTC)Don MustafaReply

Just for future reference, there was a way to ask that question, with no loss of meaning, that would have required about 95% fewer words. This advice is in your own interest. The longer the question, the greater the chance people will just switch off and ignore it (TLDNR). -- Jack of Oz [Talk] 02:29, 29 September 2012 (UTC)Reply
The question has 279 words (including "Thank you"). I'm sure we'd all love to see your 14-word version. —Tamfang (talk) 05:07, 29 September 2012 (UTC)Reply
"Is there a website that shows the candidates of each party for each European country" has 15 words. So perhaps Jack's math was of a percentage point or two. --Jayron32 05:29, 29 September 2012 (UTC)Antwort
Maths was never my strong point. I only have a Bachelor degree in it, unfortunately. -- Jack of Oz [Talk] 05:45, 29 September 2012 (UTC) Reply
(edit conflict) Wikipedia has some information for you. Start at "Politics of XXX" (where XXX is the country you are looking for). From there you can usually find a link to the most recent general election. For example, Politics of Sweden has a link to Swedish general election, 2010 which lists the results by party. Many of the countries you are asking about don't use the First past the post election system, instead using some form of proportional representation, whereby the parties are allocated seats in parliament based on the percentage of votes that party, or its slate of candidates, received in the election. For the Sweden example, Elections in Sweden covers some of the details: In Sweden, voting is done by Party-list proportional representation: voters select a party, and have the option of selecting which candidates of their prefered party they wish to serve, but they are only required to vote for the party. Many voters may not be directly aware of which candidates they are directly voting for, just the party. After the election the Sainte-Laguë method is used to allocate the number of seats to each party. The parties allocate their seats to specific candidates after the election, using in part the voting preferences of those voters who chose to give a preference. Back to Sweden, the specific "constituencies" are listed at National apportionment of MP seats in the Swedish Parliament. For the list of members of the Parliament, you can find it at List of members of the parliament of Sweden, 2010–2014. --Jayron32 02:43, 29 September 2012 (UTC)Reply

Look, the question is long, but not impossible to answer (at least in part). For Sweden, go to http://www.val.se. If you are interested in the last parliamentary election, you will find the ballots/candidate lists (not only those elected) at http://www.val.se/val/val2010/valsedlar/R/rike/valsedlar.html . Information about who got elected is and from which constituency found at http://www.val.se/val/val2010/slutresultat/R/rike/valda.html . As Jayron32 pointed out, there are cases were the order candidates elected differ from the ballot due to personal vote. You might also be interested in http://valpejl.se, where detailed biographical info on 2010 candidates is presented (however, they removed the data on unelected candidates following the election). --Soman (talk) 07:34, 29 September 2012 (UTC)Reply

For Norwegen, see http://www.regjeringen.no/nb/dep/krd/kampanjer/valg/navn-pa-alle-valglister-og-kandidater-ti/valglister-og-kandidater-20071.html?id=647367 , where all candidate lists of the 2009 parliamentary election (grouped by constituencies) are listed. --Soman (talk) 07:45, 29 September 2012 (UTC)Antwort
For Denmark, you find all candidates for the 2011 parliamentary election grouped by constituency and party listed at http://valg.im.dk/Valg/Folketingsvalg/Kandidater.aspx --Soman (talk) 07:50, 29 September 2012 (UTC)Reply

Government opposition to a private member's bill in the Parliament of the UK

Lord Dubs proposed the Succession to the Crown Bill to adjust British royal succession some years ago, but he withdrew it after the government told him that they would block it. Imagine that he had decided to be stubborn about it — how would they have blocked it? Simply by ensuring that there were enough votes against it, using whatever means they use to ensure that government bills pass? By refusing to bring it up in committee or otherwise ignoring it, so that it couldn't be voted on? Some other way? Or is there no single method that's typically used to suppress obnoxious private members' bills? Nyttend (talk) 02:12, 29 September 2012 (UTC)Reply

I'm not all that familiar with Parliament, being an American, but in the U.S. the party in charge has a lot of power in seeing what bills come to the floor. Of course, lots of bills from the minority party do make the floor eventually, but only after a lot of horse trading. However, the Majority power in Congress controls how and when bills get to the floor, so in America they weild a lot of power and could very easily prevent such bill from ever seeing the light of day. As to the UK, see Filibuster#Britain which could be one method, but I would imagine that the Government could just stonewall the bill in Parliament by any number of methods. Private_Member's_Bill#House_of_Commons_procedure mentions some options for limiting the success of such proposals. --Jayron32 02:25, 29 September 2012 (UTC)Reply
In a parliamentary system, the government is based on the parliamentary majority, and in a first-past-the-post system, as the British, it is easy for the government to achieve an absolute majority in parliament. If the government then doesn't want certain legislation to pass, given that they have the majority, it's just a matter of the voting the bill down. Jayron32 also provides an interesting comment that, potentially, the government could even block debate on the bill by preventing it 'coming to the floor'. Not knowing exactly what the procedures are for these things in the UK parliament, I am not sure how it works. To me there seems to be a contradiction in that the Speaker is chosen from the parliament itself, but is expected to be neutral/impartial. Given that s/he's been voted into office on the ticket of one party, s/he is still a partisan with certain views, even if s/he's meant to be non-partisan. First of all, given that the House of Commons votes on its speaker, it is overwhelmingly likely that the Speaker comes from the same party as the government. V85 (talk) 05:15, 29 September 2012 (UTC)Reply
Not sure what all that comment on the Speaker is about. It seems absolutely irrelevant to the question here.
At the end of the day, the government can vote down any bill it doesn't like (or indeed, support any bill it does like), no matter who proposed it. That's what "having the numbers" means. But it can also effectively block undesirable bills from ever being debated at all, because it controls what business the House deals with. This is part of the reason there so few private members' bills to begin with; and why such a small percentage of them ever go to a vote; and why such a vanishingly small percentage of them ever become law. -- Jack of Oz [Talk] 05:25, 29 September 2012 (UTC)Reply
The point about the speaker is whether the speaker, in order to support 'his/her' government could block a bill from being discussed, or could block the sponsoring MP from arguing in support of it. V85 (talk) 15:51, 29 September 2012 (UTC)Reply
The non-partisanship of the Speaker of the House of Commons (United Kingdom) is taken very seriously. On being elected, the Speaker severs all ties with their former party, and in subsequent general elections, the major parties traditionally do not field candidates in the Speaker's constituency. The Speaker (or one of the Deputies if they are presiding) only votes in the event of a tie, and this is bound by conventions (they generally vote for more debate, against amendments, and against the final passage of a bill). If a Speaker was seen to consistently favour one party over another, it could seriously damage the functioning of Parliament and might lead to some kind of constitutional crisis, so MPs are unlikely to elect a rabid partisan. Note that the current Speaker, John Bercow, was a Conservative but elected Speaker under a Labour government, while the one before last, Betty Boothroyd, was a Labour MP elected when the Conservatives had a majority, so it isn't necessarily seen as desirable to have a speaker from your own party. 81.98.43.107 (talk) 16:29, 29 September 2012 (UTC)Reply
V85, thanks for clarifying. The connection wasn't obvious until you explained it. -- Jack of Oz [Talk] 22:28, 29 September 2012 (UTC)Reply
The ability of the majority party in Parliament to effectively control every aspect of the governance of the country to its own liking is what is often called Elective dictatorship; the lack of checks on that majoritarian rule has been a criticism of Westminster-style Parliamentary democracys for centuries: it forms the core of what the U.S. constitution framers saw as Tyranny of the majority, see Federalist No. 10 where Madison argues against the sort of government that allows what he calls "majority factions" to rule by fiat without any checks and balances, a concept later revisited in Federalist No. 51. Which is not to say that, in practice, that the U.S. system we have today works any better or worse in terms of serving its people than the U.K. system does, but the criticism is in the scholarship, and in this case the derogatory "elective dictatorship" epithet comes from within the British system itself. --Jayron32 05:38, 29 September 2012 (UTC)Reply
In the US Senate, ONE Senator can say "I don't want this bill to be brought to the floor" and they will, by courtesy, not introduce the bill. 69.62.243.48 (talk) 00:49, 1 October 2012 (UTC)Reply
The key here is "Lord" Dubs proposed it. It would therefore not be debated in the House of Commons, rather, in the House of Lords. If and when that House passed it, it would then go to the House of Commons for debate - but the powers that are responsible for scheduling debates in the Commons could schedule 15 minutes for debate on it, or schedule it late on a Friday. I don't think they could refuse to schedule it at all. The House with the Power in the UK is the House of Commons, the elected house. As was seen earlier this year during debates on the Welfare Reform Bill and the Health and Social Care Bill, the House of Lords can pass all the amendments and vote against Bills passed by the House of Commons they like: at the end of the day they will be ignored. --TammyMoet (talk) 08:33, 29 September 2012 (UTC)Reply
I would imagine that the reason the government felt the need to block this particular proposal is that changes to the rules of succession need to be agreed by all the Commonwealth Realms rather than the UK alone. The Statute of Westminster 1931 forbids changes to the succession without the agreement of all 16 countries of which the Queen is Head of State. See Succession to the British throne. Changes have indeed recently been agreed at a Commonwealth summit; see Girls equal in British throne succession. Alansplodge (talk) 13:27, 29 September 2012 (UTC)Reply
WHAAOE: 2011 proposals to change the rules of royal succession in the Commonwealth realms. -- Jack of Oz [Talk] 22:05, 29 September 2012 (UTC)Reply

Pain in babies

Up to the end of the 1980s, babies underwent surgery without any anesthetic. This came to light in 1985:

Hospitalized newborns, from preemies to babies up to 18 months of age, have been routinely operated upon without benefit of pain-killing anesthesia. This has been the practice for decades but was unknown to the general public until 1985 when some parents discovered that their seriously ill premature babies had suffered major surgery without benefit of anesthesia. Up to this time, babies were typically given a form of curare to paralyze their muscles for surgery, making it impossible for them to move or make a sound of protest.
Jill Lawson reported that her premature Baby, Jeffrey, had holes cut in both sides of his neck, another in his right chest, an incision from his breastbone around to his backbone, his ribs pried apart, and an extra artery near his heart tied off. Another hole was cut in his left side for a chest tube, all of this while he was awake but paralyzed. The anesthesiologist who presided said, "It has never been shown that premature babies have pain."

The article Pain in babies seems to present this as a reasonable assumption at the time, due to the difficulty of assessing how much pain a baby is experiencing. Is there any record of a medical professional testifying as expert witness in a case of child abuse involving babies before 1985 making such claims? Has any doctor ever made such a statement under oath? It seems unlikely that they really believed this, yet kept it a secret for so long. Was there a general policy to keep this fact hidden, and if so, what was the motivation? Ssscienccce (talk) 05:50, 29 September 2012 (UTC)Reply

That argument does seem rather weak. I've heard similar arguments that "animals don't feel pain", but see no reason to believe them. Of course, there may be good arguments against using anesthetics, such as the difficulty in getting the dosage right and therefore possibility of causing harm, and the presumed lack of ability for the baby to remember the pain later. StuRat (talk) 08:31, 29 September 2012 (UTC)Reply
Please show up at my house by tomorrow. I will torture you for 6 months using the most painful methods possible. At the end, I will repeatedly knock you on the head until you suffer amnesia, so that you won't remember anything. Your post-traumatic stress disorder might also help to deepen your amnesia. --140.180.242.9 (talk) 08:54, 30 September 2012 (UTC)Reply
Yeah, I certainly don't remember the pain I felt after breaking my clavicle during birth, but apparently I just wouldn't stop crying. - Lindert (talk) 08:54, 29 September 2012 (UTC)Reply
I find it remarkable that given the number of babies undergoing medical procedures all these years, and some parents presumably asking questions about anesthesia related risks, this never came out. I hoped to read a transcript of a cross examination, I just realised cases that old won't be online. Maybe there are interviews from the time the news broke. I guess that in case the baby didn't make it through the procedure, you'd want the parents to know (think) that at least it didn't suffer during those last minutes. Ssscienccce (talk) 09:25, 29 September 2012 (UTC)Antwort
An example of the research that changed everyone's mind is Infants Feel and Remember Circumcision Pain - Study from 1997. Alansplodge (talk) 13:17, 29 September 2012 (UTC)Antwort
Having attended a bris once, which was more than enough, I can tell you the baby was screaming throughout, and I'm reasonably certain it wasn't because it was hungry or something. ←Baseball Bugs What's up, Doc? carrots14:53, 29 September 2012 (UTC)Reply
This is a pretty interesting historical question. I don't have the time to hunt around for actual references at this point, but I wonder whether part of this wasn't a balance between the negative effects of anesthesia (which can be unpredictable and difficult on small children, much less babies) with the fact of infant amnesia. The notion that babies feel no pain seems unlikely to me — it's obvious that they respond to pain stimulus. But I wonder what the actual line of thinking really was. This should be something reconstructable based on medical sources from the time. --Mr.98 (talk) 15:01, 29 September 2012 (UTC)Reply
Back at the beginning of the 90s it was still a contentious issue. This article of the NY Times discusses it. Doctors were afraid that anesthesia could suppress the blood pressure and thought that babies experienced pain differently. As a consequence, babies were just slightly anesthetized during surgery. OsmanRF34 (talk) 15:26, 29 September 2012 (UTC)Reply
  • Let me start by saying that I'm totally in favor of anesthesia for babies that undergo painful procedures. That being said, it's worth mentioning another point here: until recently, even people who were unclear on whether babies experienced pain were generally convinced that they would not remember it even if they did. That has a substantial effect on the ethical considerations. Suppose I told you that for the next hour you were going to undergo hideous pain, but afterward you would not have the slightest memory of it. That would make a big difference to you, wouldn't it? Also, for what it's worth, I think it would be reasonable to discuss some of these points at Talk:Pain in babies. Looie496 (talk) 15:49, 29 September 2012 (UTC)Reply
Right, but the problem of pain is more than just remembering how horrible it was. From the article I linked above: " bodies feel pain and react to it" and
OsmanRF34 (talk) 16:05, 29 September 2012 (UTC)Reply
Yes, right. But until recently those facts were not known. I was trying to explain the way doctors thought about it during the days when anesthesia was mostly unused in neonates. Looie496 (talk) 23:47, 29 September 2012 (UTC)Reply

I'm still puzzled by this discussion. There's an awful lot of speculation here. Where are the reliable sources telling us that babies weren't anaesthetised during surgery until around the 1990s? (I'm not interested in the other issue of whether they feel pain. From many visits to wards for premature babies, I simply know they do.) HiLo48 (talk) 23:57, 29 September 2012 (UTC)Reply

Read the article I linked to in my last contribution. Not a primary source, but it's still the NY Times. At 1992 it was still a topic open for debate. OsmanRF34 (talk) 01:04, 30 September 2012 (UTC)Reply
See PAIN AND ITS EFFECTS IN THE HUMAN NEONATE AND FETUS which states; "One result of the pervasive view of neonatal pain is that newborns are frequently not given analgesic or anesthetic agents during invasive procedures, including surgery. Despite recommendations to the contrary in textbooks on pediatric anesthesiology, the clinical practice of inducing minimal or no anesthesia in newborns, particularly if they are premature, is widespread." Although this is on the website of a pressure group, the article comes from THE NEW ENGLAND JOURNAL OF MEDICINE, Volume 317, Number 21: Pages 1321-1329, 19 November 1987. One of the sources quoted is "Shearer M H. Surgery on the paralysed, unanesthetized newborn. 1986". Alansplodge (talk) 01:18, 30 September 2012 (UTC)Reply
Thanks. That helps. But not being American, I'd be interested in the situation outside the USA too. HiLo48 (talk) 01:39, 30 September 2012 (UTC)Antwort
(EC) One more: Fetal and Neonatal Physiology: Expert Consult - Online and Print, 2-Volume Set By Richard A. Polin, William W. Fox, Steven H. Abman. It says; "Until the 1980s, newborns frequently underwent invasive procedures, including surgery, with no analgesic or anaesthetic." Alansplodge (talk) 01:47, 30 September 2012 (UTC)Antwort
That last one was also a US source, I'll have another try at a more civilized hour. Alansplodge (talk) 01:48, 30 September 2012 (UTC)Antwort
No luck on that I'm afraid. Alansplodge (talk) 14:40, 30 September 2012 (UTC)Reply

Back to the original question, the full text of the article that the OP quoted from is Standards of Practice and the Pain of Premature Infants, by Jill R. Lawson. She relates that after baby Jeffrey died, she "consulted with two attorneys. They concluded that there are no laws extant requiring doctors to mitigate pain and that unquestionably the doctor involved met 'standards of practice'." Alansplodge (talk) 14:40, 30 September 2012 (UTC)Reply

Here is a 1987 article in The Lancet (a British journal, at least historically) on the subject. I'm supposed to be able to access it through my university's subscription, but it doesn't seem to be working, so I have no idea what it says... --Tango (talk) 17:00, 30 September 2012 (UTC)Reply
There seems to be still a view in the medical profession that pain or other unpleasantness, such as the horror of being paralyzed by a drug and unable to breathe for some time while being intubated, or pain from procedures, is ok as long as the patient doesn't remember it after the operation. They routinely administer Midazolam, which makes the patient sleepy and causes amnesia. It does not particularly block pain; that is done by local anesthetics or opiods. I've had operations with Midazolam in which I remember nothing for some time before the operation, and others in which I remember getting on the table, and much of the sounds, discussion,and twinges from cutting, with Fentanyl and local anesthetics preventing the experience being worse than getting a filling by the dentist. I may be more resistant to Midazolam than the average patient. I suppose the vibration of cutting, sawing or drilling on their bodies would bother some folks in retrospect and make them less likely to go in for surgery at some later date, if there was any way to avoid the needed operation."The baby felt pain, but he will not remember it as a child or adult" or "You were in pain and distress, but you don't remember it, so it's ok" does smack of "We waterboarded you to get information, but we gave you a date rape drug so you don't remember it, so it's ok." An anesthetist told me that decades ago they were afraid of babies or small children dying under general anesthesia, because there was less of a margin for error than with adults, so pain during the operation was far preferable to explaining to the parents the child had died. Edison (talk) 17:09, 30 September 2012 (UTC)Reply


September 30

Soviet Census

How come it took the Soviet government 20 years to conduct a census after 1939, when it generally conducted censuses after a decade or so (1926, 1937/1939, 1959, 1970, 1979, 1989)? 1959 was the big exception, but I can't quite figure out why. WWII was over in 1945, but the USSR took 14 years after the end of WWII to conduct a new census. Futurist110 (talk) 00:13, 30 September 2012 (UTC)Reply

I suspect political reasons. Two that come to mind is that they didn't want to admit how many people were killed in WW2 and/or how many people Stalin had killed. StuRat (talk) 01:12, 30 September 2012 (UTC)Reply
Or practical reasons: they were killing lots of people until 59, and thought it didn't make sense to count them while they were killing. OsmanRF34 (talk) 01:15, 30 September 2012 (UTC)Reply
Neither of these are terribly compelling "theories", as both misunderstand the mindset of the Soviet bureaucracy. The Soviets had no trouble making censuses in the 1920s or the 1930s, when they were purging left and right. I doubt this sort of thing had anything to do with it; faking data would have been easy enough, and World War II created enough Soviet casualties to massage any numbers if one wanted to. My suspicion is that there are probably far more banal reasons. But there doesn't seem to be anything written about this. It would be an interesting thing for someone to investigate, but it is a non-trivial thing to try and figure out over the Internet and in English alone. --Mr.98 (talk) 01:59, 30 September 2012 (UTC)Reply
Yeah, Stalin was dissatisfied with the results of the 1937 Soviet census so he simply ordered a new one in 1939 and then rigged its results to show a larger population than it should have been. If the USSR would have wanted to manipulate some data it would have been very easy for them to do this. Also, even though the USSR was busy diverting resources to Communist takeovers and takeover attempts in Eastern Europe, China, and Korea, it would have probably still been able to have enough resources for a new census in the late 1940s/early 1950s if it desired to implement one. You're right that the reason for a lack of a Soviet census around 1950 might be something very basic, simple, and obvious. I wonder if Stalin planned to do a new census after he planned to deport the Soviet Jews to Siberia (if he had such plans in the first place). It would have been stupid for Stalin to do a new census in the late 1940s and early 1950s and then (assuming that he would have lived longer) to deport Soviet Jews to Siberia and then get blamed for a large further Jewish death toll. Of course, Stalin died before he could implement any such plans, if he had them, but had Stalin lived and done a new census after deporting the Soviet Jews to Siberia, then he could have used Hitler as a complete scapegoat for the decline in the Soviet Jewish population even if some of it would have been his fault. I speak Russian fluently, and I can read it fluently as well but very slowly. However, I speak "common" Russian, not "literary/academic" Russian, so even if I tried a search in Russian about this I won't know where to look or what search words/key words to use. Futurist110 (talk) 02:52, 30 September 2012 (UTC)Reply
The official website suggests they were busy counting other things like equipment, materials, cattle, and crops. Don't forget they had a terrible WW II, and were fully expecting to fight WW III. Zoonoses (talk) 06:10, 30 September 2012 (UTC)Antwort
Yes. The Soviet Union lost ~13.5% of its population. A large part of that from the military (i.e. from working-age people). They probably were quite busy just getting the country running again. --Stephan Schulz (talk) 13:19, 30 September 2012 (UTC)Reply
This was my initial thought, but would they really have been in such dire straits that they couldn't do a census by 1949 or 1950? It's possible, it just doesn't seem like it's obvious enough to be definitive without some evidence of them saying, "we don't have time for this." They had enough resources in the 1945-1949 time period to build an atom bomb, you'd think they could fit in a census if they were really desiring of one. --Mr.98 (talk) 14:52, 30 September 2012 (UTC)Reply

This Russian article suggests that the post-WWII famine caused the death of about a million people, and "[since] the data for that period could not be used for propaganda purposes, Stalin rejected statisticians' proposal for a new census in 1949". In any event, doing a census just may not have been viewed as a high enough priority for the country at the time. (Theoretically speaking, the USSR would not need to have decenial censuses at all, if other population counting mechanisms (birth and death registration, records of arrests and forced relocations, residential address registration) had been working 100% perfectly - although of course it was exactly the WWII period that had all these registration systems break down to various extent for several years.) -- Vmenkov (talk) 17:56, 30 September 2012 (UTC)Reply

name for a research paradigm

I'm doing qualitative research, and to satisfy various people, it is highly desirable (perhaps not essential) that I classify it as something that sounds good. It's closest to in-depth interviews, but not quite as involved. Basically I'm just finding out whether some software I've done is any good for education, and so the basic question I want to ask participants is "does it work?" It's not rocket science to assume they have a reasonable idea of its benefits, if we are talking about education (it would be different if we were talking about the effect of a drug on blood clotting, for example, or a retrospective study on pain in babies, etc etc). So it makes sense to ask people what they think, and just let them tell me, but I want to know what's the closest thing to this among official "named" research methods. I've done some searching, and every named thing I can find is quite detailed, and usually borrowed from psychology (or some other more theoretical area) and requires a lot more than just asking people for their opinions. More rigorous research will follow; this is just for a pilot. IBE (talk) 04:39, 30 September 2012 (UTC)Reply

I thought of the term "focus group", but that's used more in marketing, and involves group interviews, not one-on-one. How about just "Educational software user assessment analysis" ? StuRat (talk) 05:13, 30 September 2012 (UTC)Reply
That might in fact work, but it's safer to use an existing paradigm with a fancy sounding name and its own Wikipedia article. The corollary to whaaoe must be that if we don't, it doesn't exist. IBE (talk) 05:22, 30 September 2012 (UTC)Reply
We have articles on educational software, end-user, assessment, and analysis. Put them together and you have "Educational software end-user assessment analysis". StuRat (talk) 05:44, 30 September 2012 (UTC)Reply
Nice try ;).... assessment is a disambiguation page, and in any case, I said "an" article. Four is a bit too many. Good trick though, didn't see it coming. IBE (talk) 06:31, 30 September 2012 (UTC)Reply
I'm not sure that an editor like StuRat was trying to "trick" you, and I'd recommend being more cautious of throwing around these terms on Wikipedia. --Activism1234 06:33, 30 September 2012 (UTC)Reply

It's an evaluation. Good evaluations use a mix of methods. In the pilot phase you used simply interviews, perhaps they were semi-structured interviews, but if you only asked one or two questions then that would be too much to claim. Now as you go into the main phase you need to choose your methods carefully, not just to sound good, but to give you valid results. The most important thing you need to think about is the independence of the research from you as the software developer. Ideally you would commission an entirely independent team to do the next phase. If you can't do that, can you involve some users in designing and carrying out the research? Itsmejudith (talk) 07:18, 30 September 2012 (UTC)Reply

Not sure I can go that far, but wouldn't it be nice .... sadly, that counts as work, and I have to do it (PhD, not major software company stuff). Thanks for the terminology: nice and simple. IBE (talk) 12:52, 30 September 2012 (UTC)Reply
Then at least make the questions objective ones, like "How would you rate the ____ from 1 to 10, with 10 being the highest possible rating ?". This way, assuming you don't lie about what they said, it's not possible to "put your own spin" on the data. StuRat (talk) 16:15, 30 September 2012 (UTC)Reply
Considering how general the research method is, I'd be surprised if there was a term more specific than "survey", "poll", or "market research". Note that if you plan on asking people "does this work?", those people had better be the children who will use your software, because why would a 50-year-old know what the best way to learn elementary algebra is for a 12 year old? --140.180.242.9 (talk) 22:39, 30 September 2012 (UTC)Reply

Scottish independence

If Scotland becomes independent, would anyone from any part of the UK be allowed to move to Scotland and become a Scottish citizen, or would only people born in Scotland be allowed?

Will people from the rest of the UK wanting to jump ship and leave with Scotland be blocked from doing so?

Are there any historical precedents? — Preceding unsigned comment added by 95.25.156.221 (talk) 12:54, 30 September 2012 (UTC)Reply

There's no concrete proposal, so you'd have to guess about what the post-independence arrangements would be. The arrangements for the referendum seem to be only people living in Scotland at the time could vote (so people born in Scotland but living in say England wouldn't be); whether that applied for citizenship, no-one knows, as no-one has decided. There isn't much law about how a divorce could be conducted (the Acts of Union don't cover this eventuality) - one might look to the Dissolution of Czechoslovakia as a guideline about how amicable divorces of unified countries could be conducted (or perhaps Dissolution of the union between Norway and Sweden in 1905, although that article has less info about the internal and external legal outcomes). -- Finlay McWalterTalk 13:05, 30 September 2012 (UTC)Reply
It seems very unlikely that an independent Scotland would isolate itself from the rest of the world to the extent that British people couldn't move to Scotland. It would depend on the laws that a future independent Scotland enacts: all countries have rules regarding immigration, and there are rules about naturalisation of foreign citizens. Depending on what Scotland decided would be its relationship with the EU - whether it will be a full-blown member, or just a member of the European Economic Area - it might still have to let citizens of other European countries (including the UK) enter and work. If that's the case, then people from the UK wanting to live in an independent Scotland, that is a member of the EU, probably could move there. The only 'problem' (and I don't know if it really would be a problem for people moving to Scotland) would be whether they could find work there. Even if Scotland choses to be neither an EU member nor an EEA member, it could still choose to have lax immigration laws for UK citizens. (To me, that seems the most likely, at least in the short run.)
There are several examples of (current) states that have had referendums to break away from a larger state. One could include the East Timorese independence referendum, 1999 or the South Sudanese independence referendum, 2011, just last year. V85 (talk) 13:44, 30 September 2012 (UTC)Reply
The Scottish National Party (which is the only plausible agent for Scottish independence at present) is very Euro-enthusiastic. Whether the European Union actually wants an independent Scotland as a member is not a foregone conclusion, as they've already been told that membership won't be granted automatically, they'll have to apply like everybody else. Alansplodge (talk) 14:29, 30 September 2012 (UTC)Reply
Rather than Scotland being outside the EU and the (rest of the) UK inside, the reverse situation is a more likely scenario. Ghmyrtle (talk) 18:21, 30 September 2012 (UTC)Reply
For another possible blueprint of scission and its aftermath, consider Ireland–United Kingdom relations#Co-operation. This covers citizenship, the Common Travel Area, and various kinds of cross border cooperation and institutions. One might analogise the issue of the Treaty Ports with that of HMNB Clyde. -- Finlay McWalterTalk 16:40, 30 September 2012 (UTC)Antwort
That is a good comparison. Irish Citizens who are resident and on the electoral register can vote in UK elections where other EU nationals cannot.[3] I believe this has been the case since the founding of the Irish Free State in 1922. Alansplodge (talk) 23:17, 30 September 2012 (UTC)Reply
If you consider the Dissolution of the Soviet Union to be a relevant historical precedent, it seems most (perhaps all) post Soviet republics gave citizenship to USSR citizens resident in their territory at the moment of their independence. That's the primary factor of citizenship for at least Belarus, Ukraine, Lithuania, Estonia, and Russia. Some republics, like Lithuania and Latvia have a "law of return", and some have clauses where having grandparents from that republic is grounds for citizenship. Few allow dual citizenship (Russia does, a bit). -- Finlay McWalterTalk 17:59, 30 September 2012 (UTC)Reply
What happened when the Republic of Ireland became independent? Did people have to decide that they wanted to stay UK subjects, or was everyone living in that part of Ireland automatically considered an Irish citizen? 69.62.243.48 (talk) 00:52, 1 October 2012 (UTC)Reply
Irish nationality law#Historical provisions goes some way to answering this. It doesn't seem that people in the UK but not on the island of Ireland, who may have felt themselves to be "Irish", became or had rights to become Irish Republic nationals. It does seem that everyone involved did have the right to remain a UK national (which is quite unlike the USSR case, where people woke up one morning and found themselves solely the citizen of a country that hadn't existed the previous night, entirely without their having decided that). -- Finlay McWalterTalk 01:02, 1 October 2012 (UTC)Reply

Advertising Model of Cable and Satellite TV

How exactly do Cable TV and Satellite TV companies make money off of the advertisements for the programing that goes on television? Do they sell or lease out their channels to media companies who provide the content who then sell ad-space? Or do they do it some other way? Bakmoon (talk) 23:01, 30 September 2012 (UTC)Reply

The various television networks make money off the television advertisement that appears during the shows that they run. Some periods of broadcasting time may be dedicated to "local affiliates" which can be the local carrier, so it's complicated. These networks have various licensing agreements with the cable and satellite distributors that carry them. Premium networks like HBO negotiate higher fees which are subscribed to by the customer individually through the carrier. Other channels become available to the customer as part of a much cheaper package. All of this is negotiated between the carriers and the networks, meaning occasional changes in what is available to the viewer. But commercial revenue largely goes to the network, not the carrier. μηδείς (talk) 01:02, 1 October 2012 (UTC)Reply

October 1

Sir Anthony Knyvet

He was Governor of Portsmouth in 1544 and supervised the construction of Southsea Castle. Is this the same man that we have an stub article about: Anthony Knyvett (Black Rod)? Alansplodge (talk) 00:24, 1 October 2012 (UTC)Reply

They appear to be two different gents both named Sir Anthony Knyvett.[4][5] Clarityfiend (talk) 00:41, 1 October 2012 (UTC)Reply
Aha! Many thanks - I was getting very confused. Alansplodge (talk) 16:03, 1 October 2012 (UTC)Reply
I have now created a stub article, Anthony Knyvett (1507-1554). Many thanks Clarityfiend. Alansplodge (talk) 18:38, 1 October 2012 (UTC)Reply
You're welcome, Alansplodge. And you too, Alansplodge. Clarityfiend (talk) 23:05, 1 October 2012 (UTC)Reply
  Resolved

Views on the afterlife

The ancient Greeks believed that death was horrible, eternal, and inevitable, which seems to be an odd combination shared by nobody else (not even atheists, who probably believe death is neutral rather than horrible). How did the Greeks not go insane with such a pessimistic view, since death was very common in the ancient world? Is there any other religion that views death in this way? --140.180.242.9 (talk) 05:02, 1 October 2012 (UTC)Reply

I'm just curious. Where did you get the idea that "The ancient Greeks believed that death was horrible, eternal, and inevitable"? If you can tell us that it may give us a lead to their other thought processes (before they went insane). HiLo48 (talk) 05:17, 1 October 2012 (UTC)Reply
Not everyone had to go to Hades, some made it to the Elysian Fields. StuRat (talk) 05:18, 1 October 2012 (UTC)Antwort
Yes, where they invented baseball. ←Baseball Bugs What's up, Doc? carrots22:22, 1 October 2012 (UTC)Antwort
Baseball was invented by dead Greeks? Who knew? --Trovatore (talk) 22:27, 1 October 2012 (UTC) Antwort
Check it out for yourself: Elysian FieldsBaseball Bugs What's up, Doc? carrots00:02, 2 October 2012 (UTC)Reply
Well, the scene in the Odyssey where Odysseus calls up the ghost of Tiresias does present a pretty negative view of death. What I question is the belief that nobody else held similar views. The Navajo, as I understand it (mainly from reading Tony Hillerman books) traditionally hold a very negative view of the afterlife. Looie496 (talk) 05:25, 1 October 2012 (UTC)Reply
I wonder why we're talking as if death and the afterlife are synonymous terms. There's a lot more to a house than just the door, which is what death is in relation to the afterlife. -- Jack of Oz [Talk] 06:23, 1 October 2012 (UTC)Antwort
There is no house. Just the door. AndyTheGrump (talk) 06:39, 1 October 2012 (UTC)Antwort
The building inspectors will never approve. The makers of Blazing Saddles seem to have cottoned on to the general idea, though. -- Jack of Oz [Talk] 08:59, 1 October 2012 (UTC) Antwort
I was remembering Odysseus' conversation with Achilles in the Odyssey:
"In book 11 of Homer's Odyssey, Odysseus sails to the underworld and converses with the shades. One of these is Achilles, who when greeted as "blessed in life, blessed in death", responds that he would rather be a slave to the worst of masters than be king of all the dead. But Achilles then asks Odysseus of his son's exploits in the Trojan war, and when Odysseus tells of Neoptolemus' heroic actions, Achilles is filled with satisfaction."
Hades also seems to be a gloomy place in general. From our article: "In older Greek myths, the realm of Hades is the misty and gloomy[19] abode of the dead (also called Erebus), where all mortals go. Later Greek philosophy introduced the idea that all mortals are judged after death and are either rewarded or cursed." --140.180.242.9 (talk) 06:34, 1 October 2012 (UTC)Reply
The early Biblical notion of Sheol seems very similar to the Greek Hades. --Nicknack009 (talk) 09:46, 1 October 2012 (UTC)Antwort
In what way? Plasmic Physics (talk) 22:03, 1 October 2012 (UTC)Antwort
In the sense that it was seen as an underworld of gloom and semi-oblivion, that made no distinction between the good and the evil, and that people like Samuel could be called up from. --Nicknack009 (talk) 22:27, 1 October 2012 (UTC)Antwort
The Bible indicates that Sheol describes the grave, which is to say that both the good and the evil is called to the grave. Sheol, as used in the Bible does not refer to a place, but to the actual state of death. "Samuel" was not called up, but a deceiving spirit in disguise instead, the Bible is quite clear on the state of the dead. Only God can restore life to the dead. The Bible does not support the idea of the afterlife in any case, there is only life and death. Plasmic Physics (talk) 23:27, 1 October 2012 (UTC)Antwort
I know this is becoming a tangent to the OP's question, but Plasmic, it's not clear to me that the Samuel conjured up by the witch of Endor in 1 Samuel 28 is not in fact Samuel. The verses make it fairly plain that Saul, who knew Samuel in life, recognized him, and there is absolutely no suggestion in the passage, by Saul or anyone else, that Samuel did not in fact appear: http://www.esvbible.org/1+Samuel+28/ I'll admit that a large number of theologians have interpreted the passage as you do--that is, that it was a "deceiving spirit"--but that interpretation is not demanded by the text itself. I think it's fair to say that this passage, at the very least, suggests that Jewish ideas about death, the spirit realm, and the afterlife may have been more complicated in the era during which the books of Samuel were composed than they later have been understood through the lens of Christian exegesis (and I'm saying this as someone who identifies as Christian and takes a serious layman's interest in exegesis). Jwrosenzweig (talk) 05:57, 2 October 2012 (UTC)Antwort
If you believe that the Books of the Bible can be cross-referenced, then it becomes impossible for that subect to be Samuel. This depends entirely on whether or not you acknowledge that the Books are interconsistent. If you don't acknowledge this then my arguement is pointless. Plasmic Physics (talk) 07:08, 2 October 2012 (UTC)Antwort
I challenge your false dichotomy. I cannot see any reason to believe that the author of 1 Samuel did not think that the Witch of Endor successfully called on Samuel's spirit. And the spirit appears to give Saul accurate information, so there is no reason within the text to regard the spirit as 'deceiving'. Your assertion of 'interconsistency' is a very tall order, and an obviously unproven claim. Which other texts do you think give us reason to think otherwise?
It's not apparent that the author of 1 Samuel regards calling up a spirit as the same as 'restoring life to the dead'. There's a clear contrast with the most relevant story from Hebrew scripture, which is Elisha raising the Shunamite's son in 2 Kings 4. I also think that no credible scholar would ever claim, as you do, that 'the Bible does not support the idea of the afterlife'. New Testament examples are plentiful: I particularly direct your attention to Jesus' words to the dying thief, "I tell you truly, you will be with me in Paradise today", but also to Jesus' use of 'Abraham's Bosom' and related concepts in the parable of the Rich Man and Lazarus. And in the Hebrew scriptures, there's 2 Samuel 22:6 and Psalm 18:5 (which certainly do seem related and interconsistent), where David specifically refers to Sheol by name, and Psalm 16:10 - "You will not leave my soul in Sheol", which makes it clear that the soul is conceived of as having an existence beyond the outward life of the body, and that Sheol is a potential trap or prison for it.
It looks like you want us not just to accept that the books of the Bible can be cross-referenced, but that doing so delivers an interpretation which suits the teaching you have accepted. I submit that (a) except where the explicitly call on the reader to recall another scripture, the books of the Bible can be read in isolation, and that (b) reading them in both their scriptural and historical contexts may lead to interesting and productive insights - about Judaism, Christianity, and the ancient Near East - but that it does not lead ineluctably to the conclusions you are attempting to press on us.
For my part, I regard the Bible's teaching on the afterlife to be partial and inconclusive, but most definitely not non-existent. And I think it's fair to say that the early image of Sheol, especially as illustrated by the books of Samuel and Psalms, does have some correspondence with beliefs about the afterlife found in Mesopotamian and Greek sources from both before and after the time those stories are set, and when they were likely to have been set down in something approaching their present form. AlexTiefling (talk) 11:31, 2 October 2012 (UTC)Antwort
There is no point in discussing this with you if you don't fully understand your own premises. Plasmic Physics (talk) 23:29, 2 October 2012 (UTC)Antwort
And what do you suppose my premises are, and in what way do you think I don't understand them? Because I have a degree in this stuff, and I like to think I have a reasonable clue what I'm talking about. Alternatively, you could actually address the points I've raised, rather than attacking my intelligence or integrity. I'm concerned because the OP asked a reasonable question, and your responses so far represent fairly obvious misinformation on the topic at hand. AlexTiefling (talk) 11:17, 3 October 2012 (UTC)Reply
They do seem strikingly similar. I wonder if the Greeks could have influenced the Hebrews or vice versa in their views of the afterlife. --140.180.242.9 (talk) 23:19, 1 October 2012 (UTC)Antwort
They very well might have. The Jewish people were living in the middle of the Hellenistic world for quite a while. While the Biblical period predates much of that period, for centuries many Jewish people lived in "Hellenistic Jewish communities, and it is quite likely there was significant cross-pollenization. See also Septuagint, a major and widely used Greek version of the Hebrew Bible. Again, while Hellenism occurred too late to be considered during the times the Hebrew Bible describes, later Jewish traditions would certainly have not been immune to such ideas, being in such close contact with the Greek world. --Jayron32 06:11, 2 October 2012 (UTC)Antwort
There were some Greek influences, but they should not be over-exaggerated. Before Alexander's conquests, Judea was a kind of minor inland province, somewhat insulated from sea-trading, and not quick to embrace foreign theological concepts. The scribes and priests of the Jerusalem temple, or the Babylonian exile, would have had zero knowledge of or interest in Greek literature. Even for a few books of the Hebrew Bible where it's widely considered possible that they were written after Alexander, and that there may be significant general overall Greek influences -- such as "Ecclesiastes" -- it's still extremely difficult to point to any particular passage as being specifically influenced by a passage in any Greek work, and there's a distinct paucity of Greek loanwords... AnonMoos (talk) 11:00, 2 October 2012 (UTC)Antwort
Which is why I stated exactly what you stated. When I said "Again, while Hellenism occurred too late to be considered during the times the Hebrew Bible describes..." what I meant by that was "Hellenism occurred too late to be considered during the times the Hebrew Bible describes" I'm sorry if that was unclear. --Jayron32 14:02, 2 October 2012 (UTC)Antwort
OK, but the classical concept of "Sheol" is contained in the parts of the Bible which date from too early for meaningful Greek influence, while during the Hellenistic period many (not all) Jews moved towards an apocalyptic resurrection and heaven-vs.-hell view of the afterlife (which is generally ascribed mainly to Persian influence, if any outside influences were involved), so I'm not sure when significant Greek influence on Jewish views of the afterlife could have occurred... AnonMoos (talk) 14:24, 2 October 2012 (UTC)Antwort
Which is not to say that the Persian and Greek and later Hebrew concepts didn't come up in the same cultural millieu. These cultures had constant continuous contact over many centuries, and it would be surprising if they all didn't have profound impact all on each other. The Achaemenid Empire dates from the founding of Zoroastrianism, which happened at the same time as the Babylonian captivity period when much of the Hebrew Bible was penned (though it describes earlier periods, it was written down at this time) and this is also the time Classical Greece reached its Zenith and when much of Greek Mythology was committed to its classic form. So, you have a case where three major religious traditions are being composed during the same (roughly) overlapping 200 year period, among cultures which are in constant contact and yet it is supposed to be surprising that there are parallels between Greek Hades and some concepts of Sheol, as well as the "Heaven-and-Hell" aspects of Zoroastrianism? I'd say I would be shocked if such connections weren't made, given the historical realities. --Jayron32 17:22, 2 October 2012 (UTC)Antwort
There were many possible paths of cultural influence; however, it's hard to get around the facts that before 332 BC there was no meaningful direct influence of Greek mythology or literature on Biblical Judaism, while after 332 BC, Jewish views of the afterlife tended to move away from a Hades-like conception... During the era when Greek influence became more historically plausible, Jewish conceptions of the afterlife actually became less Greek-like. AnonMoos (talk) 19:15, 2 October 2012 (UTC)Antwort
I think it's quite widely accepted that early Greek religion and mythology were influenced by the Phoenicians, aka Canaanites, who also influenced the early Israelites. They're all nations on or not far from the coast of the eastern Mediterranean, so some degree of cross-pollinisation would be unsurprising. --Nicknack009 (talk) 11:02, 2 October 2012 (UTC)Reply

Never give a sword to a man who cannot dance

One of my friends told me that this was a quote by Confucius, as a number of quote sites online mention. I feel that this is spurious however, and I couldn't find anything in the Analects that says anything to this regard. What's the origin of this quote, or is this just the work of some fortune cookie hack? bibliomaniac15 07:26, 1 October 2012 (UTC)Antwort

Never take a quote from a man who cannot provide the source. ;-) AndyTheGrump (talk) 07:30, 1 October 2012 (UTC)Reply
I checked two different translations of the Analects[6][7] and the words "sword" and "dance" do not appear in both of them. For what it's worth this Chinese guy[8] made the exact same observation as you. A8875 (talk) 07:50, 1 October 2012 (UTC)Reply
The digging I did on the interwebs suggests this quote is from a work of James Macpherson, although the sites I found this claim on were too unreliable to the point of even being worth mentioning. Specifically this quote was supposed to originate with Fionn mac Cumhaill as written by Oisín and translated by Macpherson, although it's worth noting that Macpherson made up pretty much everything he "translated". Someguy1221 (talk) 08:50, 1 October 2012 (UTC)Antwort
I'd be surprised if you can find it recorded much earlier than modern mythologizer Robert Bly's Iron John (book) who says "Michael Meade reminds us of the old Celtic motto: "Never give a sword to a man who can't dance."" meltBanana 13:00, 1 October 2012 (UTC)Antwort
When I saw this topic in the TOC I immediately thought of Scottish sword dances before I even scrolled down to read this. Roger (talk) 14:20, 1 October 2012 (UTC)Antwort
From memory, there's some sort of famous "sword dance" written by Scriabin oder Rimsky-Korsakov oder Kabalevsky or someone like that.--Shirt58 (talk) 14:53, 1 October 2012 (UTC)Antwort
Cancel that. Was thinking of the "Sabre Dance" from Khachaturian's Gayane.--Shirt58 (talk) 15:15, 1 October 2012 (UTC)Antwort
I thought of sword dancing too, but I think the point is, that if you're not coordinated enough to be able to dance properly, then you're likely to be a worse-than-useless with a sword. I'm probably stating the obvious though. Alansplodge (talk) 21:15, 1 October 2012 (UTC)Reply
Philosopher Wilson called it an ancient Celtic saying that goes, "never give a sword to a man who can't dance." Rich Goodhart has a CD, "Never Give A Sword To A Man Who Can't Dance." Choreographer Joe Chvala mentions a Celtic motto, "Never give a sword to a man who can't dance," to emphasize the mythically destructive and creative duality of aggression. Similarly, he explains how the berserks worshipped Odin, the Norse god of war and poetic inspiration, "which are two things people don't generally put together, but the dance does." Another reference mentions, old Celtic saying: "Never give a warrior a sword before he learns to dance". Another reference mentions, "There is an old Celtic proverb that says, "Never give a sword to a man who can t dance." In other words, don t entrust weapons to a man who is not also capable of experiencing joy and passion." Another reference, "There is an ancient eastern saying, "Never give a sword to a man who can't dance, or the Celtic saying, You cannot pick up the sword until you have picked up the drum."[9]. -- Uzma Gamal (talk) 05:28, 2 October 2012 (UTC)Reply
Both 'Celtic' and 'Eastern' are extremely general terms. Can you be more specific, and do you have sources? AlexTiefling (talk) 11:04, 2 October 2012 (UTC)Reply

anti-zionism

If people in Israel are anti-zionist, why don't they leave Israel instead of staying there? as a matter of fact why doesn't Israel kick out the anti-zionists? — Preceding unsigned comment added by 70.31.16.44 (talk) 14:56, 1 October 2012 (UTC)Reply

Do you have any idea what the words you're using mean? -- AnonMoos (talk) 15:25, 1 October 2012 (UTC)Reply
I assume that last word in their post was meant to repeat "anti-Zionist". StuRat (talk) 22:37, 1 October 2012 (UTC)Reply
The last sentence in its original form as of 14:56, 1 October 2012 is incoherent semi-gibberish... AnonMoos (talk) 10:36, 2 October 2012 (UTC)Reply
Yes, they since corrected the last word from "zionist" to "anti-zionist". StuRat (talk) 18:52, 2 October 2012 (UTC)Reply
According to the Oxford Dictionary of English, 'Zionism' means a movement for (originally) the re-establishment and (now) the development and protection of a Jewish nation in what is now Israel. The opposite (anti-zionism) would be to oppose the existence of a Jewish nation, which is in essence what Hamas strives for (the annihilation of Israel). It makes sense to me that Israel would not consider that ideology particularly constructive. - Lindert (talk) 15:50, 1 October 2012 (UTC)Reply
I'm not sure what argument the OP is trying to make. Roughly, and very much generalizing, you could divide anti-Zionists in Israel in three categories: 1) Palestinians who hold Israeli citizenship, who have roots in the land and who have no intention of leaving. Many of them would prefer a secular state, a state with equality for all citizens. They were there before the State of Israel was conceived, and any expulsion would be fiercely resisted 2) A minority within the Jewish left, who likewise would prefer a secular state. Most of them would be children or grandchildren of migrants to Palestine/Israel. They generally thread a fine line balancing themselves between anti-Zionist and the fact that their own families are residents of Israel through Aliyah (a pillar of Zionist doctrine). 3) some ultra-orthodox Jews (like Neturei Karta), who don't support the notion of a Jewish state. The latter two categories are very small. Any draconian expulsion (as the OP seems to want to imply) would have no benefits and create many, many problems for Israeli policy-makers.
And @Lindert, no anti-Zionism is not about 'annihiliation', it is a critique of a chauvinist doctrine. --Soman (talk) 16:42, 1 October 2012 (UTC)Reply
And what 'chauvinist doctrine' may that be? If someone can support 'the development and protection of a Jewish nation in what is now Israel' (zionism) and at the same time oppose some 'chauvinist doctrine' (supposedly anti-zionism), does that mean that one can be a zionist and an anti-zionist at the same time? - Lindert (talk) 17:08, 1 October 2012 (UTC)Reply
Soman is saying that Zionism is the chauvinist (bellicose patriotism) doctrine. I don't see any way you can be both Zionist and anti-Zionist at the same time. You're either for an officially Jewish state or you aren't. Opposing some parts of it, but still wanting an officially Jewish state, is just another form of Zionism. It isn't anti-Zionism. Soman is correct in pointing out that there are plenty of anti-Zionist critiques which aren't based on "annihilation of Israel".--Mr.98 (talk) 18:01, 1 October 2012 (UTC)Reply
The annihilation of a Jewish nation could simply mean the annihilation of the state's discrimination against non-Jews. I oppose "the development and protection of a Christian nation in what is now America", but only because I think American policies (including immigration policies) should not discriminate against any religion, not because I want New York to go up in flames. --140.180.242.9 (talk) 18:16, 1 October 2012 (UTC)Reply
The term "annihilation" is obviously one loaded with violent connotations — it means "complete destruction." There are no doubt some groups out there who want Israel and the Jewish people annihilated, but they should be considered the exclusive definition of what anti-Zionism means. --Mr.98 (talk) 22:20, 1 October 2012 (UTC)Antwort
I think you're missing a negation in the last clause. --Trovatore (talk) 02:44, 2 October 2012 (UTC)Reply
I find elements of the above discussion problematic in their use of the key terms - including the OED definition. Kindly consider:
  • The Jewish "people" - a better term than "nation" as the latter is usually associated with geopolitical locales - can be seen as including (a) adherents of the Jewish religion (b) individuals who descend from Jewish forebears and who either (b1) do or (b2) don't identify with this ethnicity apart from religious belief/practice as in (a). Almost all (a) are also (b), whereas many (b1) and even more (b2) are not (a).
  • The state of Israel was established to be a homeland for the Jewish people (not "a" Jewish "nation"), and those who qualify per the Israeli Interior Ministry are granted residence and citizenship. This preferential treatment is considered by Zionists a goal worthy of continued support due to antisemitism in other sovereign nation-states. Not incidentally, Israel is the world's only country (and this only since 1948) to institute what I'll call Jewish culture: the Hebrew language, the Jewish calendar of holidays, etc.
  • The state of Israel doesn't operate entirely on Jewish religious law, though this is enforced in many areas of life in the domain of the abovementioned Ministry of the Interior.
  • "Orthodox" and "ultra-Orthodox" Jews are numerically a minority, though their political parties receive disproportionately high support due to the clout they wield in coalition system of government.
  • Minorities do have government-funded support, notably a separate education system for the Arab population (language of instruction: Arabic).
For the purposes of this query, looking only at "anti-Zionists living in Israel": they oppose the privileges granted to Jews and would prefer a "nation[-state] of all its inhabitants/citizens." (A separate case from some believing Jews who maintain that only the Messiah can bring about a Jewish sovereign state on Earth.) Perhaps the manifesto of some anti-Zionist party or similar organization would specify whether Judaic religious practices would be included along with others, i.e. Muslim, Christian, or secular (similar to the U.S.A.'s "separation of Church and State"), or be relegated to minority status (and possibly delegitimized) were the Jewish/Zionist hegemony replaced by either a secular, egalitarian multi-stream, or non-democratic form of government. I can also state that it's possible to support the Zionist call for Israel as a homeland for the world's Jews while demanding - as written in the Basic Law amendments (notably that of Human Dignity and Liberty) to Israel's Declaration of Independence - equal treatment/opportunities/rights/responsibilities for all its inhabitants, perhaps on the model of other multilingual, multiethnic democracies. -- Deborahjay (talk) 12:10, 2 October 2012 (UTC)Reply
Funny thing how the Arabic word for "Arab nationalism" (قومية) literally means "tribalism" (as expressed and exemplified in such literary masterworks as Three Whom God Should Not Have Created: Persians, Jews, and Flies), while the country of Malaysia has an overtly and explicitly racist governing philosophy backed up by an overtly and explicitly racist constitution, yet for some reason Soman never seems to criticize those nationalisms... AnonMoos (talk) 11:19, 2 October 2012 (UTC)Antwort
If this is some kind of personal quarrel you have with Soman, I don't want to get involved. But this discussion is about Jews and Zionism, not Arabs, Malays, or their constitution. Soman also didn't denounce Stalin in this thread, but that doesn't imply anything about his beliefs about Stalin. --140.180.242.9 (talk) —Preceding undated comment added 20:58, 2 October 2012 (UTC)Antwort
It's not a personal quarrel as such, just that I'm a little tired of the trotting out of pseudo-leftist jargon buzzwords (such as "chauvinist" here) as some kind of apparent substitute for reasoned critical thought when it comes to certain topics (as for Stalin, see the use of buzzword "progressive" at Wikipedia:Reference desk/Archives/Humanities/2011 November 5#Israel & American policy...). -- AnonMoos (talk) 23:24, 2 October 2012 (UTC)Reply

October 2

What makes a country "independent"? (e.g. Curacao)

This Associated Press article about the Caribbean island of Curacao says near the bottom Curacao is an independent country [emphasis mine] within the Kingdom of the Netherlands and the governor is the representative of the Dutch monarch. That sounds to me pretty close to saying that, say, Australia is an independent country that is a Commonwealth realm within the Commonwealth of Nations, with a shared queen. But I looked up Curacao in Wikipedia, and it seems to be more complicated than that. It says in the Politics section The Dissolution of the Netherlands Antilles came into effect on 10 October 2010.[32] Curaçao became a country within the Kingdom of the Netherlands, with the Kingdom retaining responsibility for defence and foreign policy. The Kingdom was also to oversee the island's finances under a debt-relief arrangement agreed between the two. The article Politics of Curacao says Curaçao has full autonomy on most matters, with the exceptions summed up in the Charter for the Kingdom of the Netherlands under the title "Kingdom affairs". Here and elsewhere Wikipedia conspicuously avoids calling it an "independent country". And Curacao does not appear in Member states of the United Nations, which says "In principle, only sovereign states can become UN members".

So how does one decide whether to refer to a country like Curacao as "independent" (or for that matter "sovereign"--"sovereign" is especially vague since it is used in referring to American Indian tribes and American states--there are degrees of sovereignty.) The statement with the Kingdom retaining responsibility for defence and foreign policy [of Curacao] sounds to me just like this statement from our article associated state in the table with reference to Palau, Federated States of Micronesia, and Marshall Islands, all of which are in the UN: United States provides defense, funding grants and access to U.S. social services for citizens of these areas under the Compact of Free Association.

So given that there presumably is no official definition of "independent country", where does one draw the line between "independent" and "autonomous", or "independent" and "not independent"? And where does the UN draw the line between sovereign enough to be a member and not sovereign enough? Duoduoduo (talk) 16:52, 1 October 2012 (UTC)Reply

A starting point when it comes to defining an 'independent state' is the Montevideo Convention which states that a country must have a 1) population, 2) territory, 3) government, and 4) ability to enter into diplomatic relations with others. From my own studies, my teacher emphasised that the country would have to have control of its territory, so an occupied country won't be seen as independent. When it comes to the UN, the other countries have to vote on them becoming members. This is why Taiwan is unlikely to become a UN member, as the PRC will block it (and when Taiwan held China's UNSC seat, it blocked Mongolia's membership, as it considered Mongolia to be part of its traditional territory). Similarly, the problem for Palestine is that the US will block its independence from Israel (so long as Israel doesn't give the go ahead). Although the list of members of the UN is usually seen as the list of sovereign states, there is no legal requirement for states to be members of the UN. The famous example is Switzerland that for a long period chose not to be a UN member, although no one would contest the fact that it's an independent state.
I think that when it comes to actually assessing whether a state is independent or semi-independent, one has to look at the facts on the ground: Is it self-governing and in charge of its own territory, even if that territory is claimed by someone else? In a case such as Curacao, I guess that we just accept the treaty between the Netherlands and Curacao, i.e. so long as they consider themselves to be independent, but with the same King, we accept that as being their relationship. V85 (talk) 19:41, 1 October 2012 (UTC)Reply
Another factor, not unrelated to UN membership but still separate from it, is recognition by other countries. That is, recognition of the "Government of Curaçao", whatever that means, as legitimate and separate and autonomous and different from the Government of the Netherlands. Does Curacao claim this status, and does any other country recognise it? -- Jack of Oz [Talk] 20:19, 1 October 2012 (UTC)Antwort
Britain has a consulate there, rather than an embassy, which I think means it doesn't recognise it as an independent sovereign entity. Our article says: "Curaçao became a country within the Kingdom of the Netherlands, with the Kingdom retaining responsibility for defence and foreign policy." so that isn't surprising. --Tango (talk) 21:06, 1 October 2012 (UTC)Antwort
Britain has multiple embassies and consulates in Japan, so I doubt that actually has anything to do with it. The British Gov also has a High Commission, a Deputy High Commission, two High Commission Liaison Offices, and another Liaison Office in Nigeria. Having an embassy in a country does not mean that they recognize the country as a sovereign entity. KägeTorä - (影虎) (TALK) 06:32, 3 October 2012 (UTC)Antwort
Maybe the deciding factor, when a country's defense is in someone else's hands, is whether the country can unilaterally cancel that defense agreement (as say Palau could) or whether it cannot do that unilaterally (as perhaps is the case with Curacao?). Duoduoduo (talk) 22:11, 1 October 2012 (UTC)Antwort
I don't think so, Monaco's defense is managed by France but I would call it independant. It's foreign policy is independent though, which allowed it to become a UN member. --85.119.27.27 (talk) 09:40, 2 October 2012 (UTC)Antwort
I agree, foreign policy is far more important than defense when it comes to independance. Plenty of small countries have agreements where a larger country provides their defense since it wouldn't be practical or efficient for them to maintain their own standing army. That doesn't stop them being independant. --Tango (talk) 11:06, 2 October 2012 (UTC)Antwort
Sure, as I said countries like Palau are independent while revocably handing over their defense to a larger country. I assume Monaco is in that category--it could decide that henceforth France no longer is responsible for its defense (or is that not right?). But my suggestion was that a country that by law cannot revoke its defense agreement is therefore not independent. Comments? Duoduoduo (talk) 14:03, 2 October 2012 (UTC)Reply
The article on Dependent territory is probably also relevant here, with many examples showing different degrees of "dependency". Ghmyrtle (talk) 09:54, 2 October 2012 (UTC)Reply

creative common licensing for the work already in public domain

Is it possible to license my art work, which is currently in public domain, under creative commons? Will it prevent others from for-profit implementations? Should I prove as it is my work? --V4vijayakumar (talk) 04:09, 2 October 2012 (UTC)Reply

Good question, wrong noticeboard. Take this exact same question, and ask it again at Wikipedia:Media copyright questions. --Jayron32 04:12, 2 October 2012 (UTC)Antwort
thanks. --V4vijayakumar (talk) 04:15, 2 October 2012 (UTC)Reply

About Indian independence fighter in INA malaysia

Could I please get the currnet deatils about Mrs.Janaky Athi Nahappan, Malaysia? Who fought for Indian independence through Indian National Army of Subash Chandra Bose. — Preceding unsigned comment added by 117.193.98.241 (talk) 04:16, 2 October 2012 (UTC)Reply

We have an article about Janaky Athi Nahappan. Looie496 (talk) 06:20, 2 October 2012 (UTC)Reply

Did communist forces use civilians as cannon fodder in Menglianggu Campaign?

Just mentioned an excerpt form Xin Haonian in zh:人海战术 says "during the Menglianggu Campaign, in order to eliminate the 74th Reorganized Division led by Zhang Lingfu, the communists forced landowners, rich farmers, their children and nakesd women to be their cover."(在孟良崮戰役中,為了消滅抗日名將張靈甫的整編第74師,中共竟逼迫地主、富農及其孩子和裸體婦女在前面做衝鋒掩護。) but I cannot find such saying anywhere else in the Chinese and English article of Menglianggu Campaign. Can that claim ever be true?--Inspector (talk) 06:07, 2 October 2012 (UTC)Reply

"nakesd " ? StuRat (talk) 07:10, 2 October 2012 (UTC)Reply

The role sounds like what would be called "human shields", not "cannon fodder"... AnonMoos (talk) 10:34, 2 October 2012 (UTC)Reply

We have an article; Human shield. Alansplodge (talk) 12:10, 2 October 2012 (UTC)Reply
That is right. But the Human shield article says nothing about even the Chinese Civil War.--Inspector (talk) 13:02, 2 October 2012 (UTC)Antwort
True; but I thought it might be useful if the article is updated. A quick look at Google didn't give any useful information in English unfortunately. Alansplodge (talk) 14:36, 2 October 2012 (UTC)Antwort
Perhaps if it will be better to ask among those who studied this area. But I would like to know what is the westarn view about the tactics employed during the battles in Chinese Civil War?--Inspector (talk) 05:41, 3 October 2012 (UTC)Reply

I studied a lot of mid-century Chinese politics, war and military affairs, and this incident is not one I’ve come across. Still, it may have simply been under-reported in scholarly works. More to the point, my first thought is “who is the source?” That will often tell you all you need to know about the likely accuracy of a one-off report. ADD: The source is a pro-Nationalist (KMT) newspaper in Shanghai, in 1946. That puts it in the grey area between heavily censored news and deliberate propaganda. DOR (HK) (talk) 09:26, 3 October 2012 (UTC)Reply

Karaite Judaism vs. Fundamental Christianity

What's the difference between the two? What are the similarities? As far as I can tell, they both seem to worship the Bible - the former the Hebrew Bible, the latter the Christian Protestant Bible. 140.254.227.46 (talk) 13:40, 2 October 2012 (UTC)Reply

Leaving aside the obvious differences between Christians and Jews, I think that if you honestly believe any group committed to the literal truth of the Ten Commandments can really be said to worship a book, you have not understood these religions. AlexTiefling (talk) 13:42, 2 October 2012 (UTC)Reply
The article to read is Fundamentalism which explains fundamentalist strains of all religions in broad terms, and will help the OP to fix the misconceptions they appear to be working from (if indeed, they are interested in fixing their own misconceptions). --Jayron32 14:00, 2 October 2012 (UTC)Antwort
Given some of the OP's other edits, they may well not be. AlexTiefling (talk) 15:06, 2 October 2012 (UTC)Antwort
Sorry. I'm using a public computer. 140.254.227.46 (talk) 15:21, 2 October 2012 (UTC)Reply
Not sure whether Karaites are very meaningfully "fundamentalist" but they're strictly scripturally literalist -- applying the sola scriptura principle in a more thoroughgoing way than Protestants have traditionally done, to the degree that they've sometimes been considered effectively a different religion than Rabbinic or "Rabbanite" Judaism... AnonMoos (talk) 14:05, 2 October 2012 (UTC)Reply

While the difference between Jews and Christians is pretty obvious (happy to elaborate if necessary...), I think a more interesting question would be the difference between Karaites and Sadducees... I don't know that myself. --Jethro B 00:40, 3 October 2012 (UTC)Antwort

Don't think there's any real historical continuity between the two, but both vehemently rejected the "Torah she-be-`al peh"... AnonMoos (talk) 08:52, 3 October 2012 (UTC)Reply

Historical Violence

So I've read information in the past that addressed that we live in the most peaceful time in history. I couldn't find a solid article here that addressed violence today in comparison to violence in recent or even ancient history. Does that exist, or where would I find more good sources comprehensively looking into violence rates in the past. Thanks Chris M. (talk) 14:02, 2 October 2012 (UTC)Reply

Not fully comprehensive (since comprehensive historical information often just isn't available), but see The Better Angels of Our Nature for a prominent recent book... AnonMoos (talk) 14:29, 2 October 2012 (UTC)Reply
(Edit Conflict) A quick Google under "Do we live in more violent times?" has New Scientist - Steven Pinker: Humans are less violent than ever as the first result. Alansplodge (talk) 14:33, 2 October 2012 (UTC)Reply
It should be noted, I suppose, that Pinker's conclusions are considered fairly controversial. He more or less ignores the World Wars, for example, as anomalies to the general trend. More than a few people have taken issue with that sort of thing. --Mr.98 (talk) 15:29, 2 October 2012 (UTC)Reply
I suppose once you dismiss a few tens of millions of deaths as anomalies, you can come to any conclusion you want.--Wehwalt (talk) 17:19, 2 October 2012 (UTC)Reply
Even if you count them, they aren't as significant, relative to a global population of billions, as you might think. StuRat (talk) 18:50, 2 October 2012 (UTC)Reply
It also depends on what you call 'now'. Now in the XXI century we enjoy one of the most peaceful times of humankind, but in the last 100 years not so. The compelling reason to define 'now' in a more limited way is certainly that that is what matters to us. OsmanRF34 (talk) 19:42, 2 October 2012 (UTC)Reply
List of wars by death toll could give one a start. Collect data by years, compare to world-wide population during those times, and see when the greatest proportion of the world population died in wars. An idea on methodology. The incompleteness of the Wikipedia article is probably a hindrance, but if one could get the data from better sources, I'd think that's how one would do it. --Jayron32 19:59, 2 October 2012 (UTC)Antwort

A Winged Horse

My understanding is that Pegasus is the name of the winged horse ridden by Bellerephon, child of Medusa 'n' all that. My question is did the Greeks have a word for a generic winged horse? People nowadays just call any horse with wings, 'a pegasus' but I'm curious if there exists a word that the Greeks may have used.24.218.57.201 (talk) 16:36, 2 October 2012 (UTC)Reply

Presumably they just called Pegasus the ancient Greek equivalent of a "winged horse". If there were more than one, they might have had a name for them collectively, like Gorgons for Medusa and pals. StuRat (talk) 18:41, 2 October 2012 (UTC)Reply
Pegasus is the name of a mythological winged-horse character. A theoretical word formed from ancient Greek roots meaning "winged horse" would be something like Pterippos πτεριππος... AnonMoos (talk) 19:26, 2 October 2012 (UTC)Reply
Not one word, but Apollodorus calls him "hippos ptenos". Adam Bishop (talk) 21:52, 2 October 2012 (UTC)Reply
"hippos pteros", I should think, as with the Pterosaurs. ←Baseball Bugs What's up, Doc? carrots02:06, 3 October 2012 (UTC)Reply
Πτερος is the noun meaning "feather" or "wing" suitable for forming compound words, but Πτηνος is the adjective meaning "feathered" or "winged" suitable for use in multi-word phrases... AnonMoos (talk) 08:42, 3 October 2012 (UTC)Reply

Was there more than winged horse in Greek mythology? Alansplodge (talk) 12:04, 3 October 2012 (UTC)Reply

I assume you're trying to ask if there was more than one winged horse. There were two, Pegasus and his twin brother Chrysaor. I don't know of any others in the standard canon. --Jayron32 12:33, 3 October 2012 (UTC)Antwort

Criminal law in a US private bill

Private Law 95-10, signed by US President Carter in late 1977, directed the Postal Service to forgive some debts and the Treasury Department to repay money that had already been paid on the debt. It contained a provision stipulating that no more than a certain percentage of the amount repaid could be devoted to attorneys' fees, and the subsection concluded with "Any person violating the provisions of this subsection shall be guilty of a misdemeanor and upon conviction thereof shall be fined in any sum not exceeding $1,000." Imagine that someone had violated these provisions: what would the charge have been? Is there some part of the US Code that gives a default name to crimes that aren't named by the statutes that create them? I don't remember seeing any kind of criminal law in a private bill before. 2001:18E8:2:1020:81A9:10F5:10B3:3C4A (talk) 17:04, 2 October 2012 (UTC)Reply

Possibly Contempt of Congress. Certainly it would only have bound the parties affected, who would be on notice of it.--Wehwalt (talk) 17:16, 2 October 2012 (UTC)Antwort
It has nothing to do with that. Shadowjams (talk) 02:18, 3 October 2012 (UTC)Reply
They can just say "You are charged with violating statute X, subsection Y, of ...". Even when crimes have names, they often include a variety of offenses, so it's important to specify the exact sections under which they are charged.StuRat (talk) 18:45, 2 October 2012 (UTC)Reply
Who are these they that can charge you with things? I seem to remember Al Gore saying his crimes were unpunishable because there existed no controlling authority to prosecute them. μηδείς (talk) 20:05, 2 October 2012 (UTC)Antwort
What? Shadowjams (talk) 02:18, 3 October 2012 (UTC)Reply
Well, if Congress passes a law and doesn't pass the responsibility for trials onto anyone else, then it would be up to them to press charges. It's not very practical, but is possible. They generally only preside over impeachment trials. StuRat (talk) 21:23, 2 October 2012 (UTC)Reply
If you look at most indictments they list the statute itself, or actually more technically they list the code. There's no necessity to have a law in the code. Specifically see U.S. Code#Uncodified statutes. Private laws are not codified, but they still have the same force of law. There's no need to talk about Contempt of Congress (which is quite different) or worry about who would prosecute (the U.S. Justice Department). Shadowjams (talk) 02:18, 3 October 2012 (UTC)Reply

Line of Descent from David

There were two monarchies in history that claimed descent from King David, Ethiopia and Georgia. Do any of these monarchies have a direct genealogical line of descent in their tradition from David down to the present? David IV of Georgia is called the 78th descendant of King David so there must be some line to go by.--The Emperor's New Spy (talk) 18:05, 2 October 2012 (UTC)Reply

The Wikipedia articles Origin of the Bagratid dynasties and History of Armenia (Movses Khorenatsi) has some information; apparently Moses Khorenatsi has geneologies for the Bagaratid (Georgian and Armenian Royal Dynasties) going back to Adam, so that may give you some place to look for that. The Wikipedia article on the Bagrationi dynasty of Georgia also cites Sumbat Davitis-Dze and others as being a source for that genealogy. Some leads to follow. The article Solomonic dynasty has information on the Ethiopian dynasty, but it is sadly lacking in background and references. --Jayron32 18:25, 2 October 2012 (UTC)Antwort
When we're talking about lines of descent from Adam, most scientific scholars dismiss at least parts of those as fiction. So, the question comes up as to whether any genealogies from David to now are reliable. StuRat (talk) 18:56, 2 October 2012 (UTC)Reply
Probably not any more so. Claiming "decent from David" isn't much more reliable than how many Germanic and Norse dynasties claimed decent from Odin, with the exception that David was probably an historical figure. Genealogies of David after the Babylonian Captivity are covered at Exilarch and Davidic line, the last article contains the salient line "In general, the validity of such claims — as that of most claims to royal descent after a considerable passage of time — is difficult to check."--Jayron32 19:09, 2 October 2012 (UTC)Antwort
I understand the original question to be "is there a list of descendents in order", i.e. do these families have an entry for someone in each generation, or do they just have a legend saying "King ___ who founded this dynasty was the xxth descendent of Azariah son of Amaziah, but we don't know the names of everyone in between"? If I understand it correctly, the original question really isn't asking about the historicity of these accounts. 66.244.68.64 (talk) 20:54, 2 October 2012 (UTC)Antwort
Yes, this isn't a question about the historic truth behind the Davidic line. It is asking if there is a complete line in cultural reckoning?--The Emperor's New Spy (talk) 20:56, 2 October 2012 (UTC)Antwort
At Wikipedia no, but above I gave what Wikipedia articles indicate to be sources that likely do, themselves, have such a list. You would probably start looking at those original historic sources, or translations thereof, for such information. --Jayron32 21:27, 2 October 2012 (UTC)Antwort

Here's a third monarchy that claimed descent from King David. --Dweller (talk) 21:50, 2 October 2012 (UTC)Reply

Jayron32 wrote - "Claiming "decent from David" isn't much more reliable than how many Germanic and Norse dynasties claimed decent from Odin, with the exception that David was probably an historical figure." I can understand where Jayron32 is coming from. However, interestingly enough, I actually have a friend whose family throughout the generations kept a family tree with their descendants, and it goes all the way to David. Of course, it could be fabricated, but I don't think that's likely. A lot of families keep family trees throughout the generations, and his family is one of them. It's pretty interesting. Anyway, I don't know much about the actual question, but if you're wondering about the alleged descent from David as written in the Bible (and hence the alleged descent from Adam, as that can be traced in Genesis), then Books of Chronicles gives that whole thing. Of course, whether you want to believe it or not is up to you - I'm just offering it if you want to see it from a Biblical standpoint. Hope it helps. --Jethro B 00:35, 3 October 2012 (UTC)Antwort

I think it is highly likely that it is, at least in part, fabricated or, at least, contains innocent mistakes. You're talking about over a 100 generations spanning thousands of years. Do you really think it's going to be flawless? Do you think nobody had an affair and pretended the resulting child was their husband's? Or that there were no gaps when nobody was filling it in and then it got filled in afterwards with someone's best guesses? That they were descended from David isn't really in doubt - when you're talking about someone that long ago, either they have no living descendents or almost everyone (at least everyone with European/Middle-Eastern descent) is their descendant (see identical ancestors point). --Tango (talk) 11:43, 3 October 2012 (UTC)Reply

Representation in the Dutch Senate and House of Representatives?

Do the special municipalities of Bonaire, Sint Eustatius and Saba get any official representation in the Dutch Senate or House of Representatives? (Or anywhere else in the government of the Netherlands, for that matter?) --CGPGrey (talk) 18:51, 2 October 2012 (UTC)Reply

I don't believe so; looking around the best information I can find is at Caribbean Netherlands which indicates that the "National Office for the Caribbean Netherlands" provides all liaison between the islands and the National Government; but I can't find any information to indicate direct representation in the States-General of the Netherlands. However, the States-General article lists a connection to the template "National Legislative Bodies of the Americas" indicating some connection there. Of course, the States-General doesn't appear to be elected from single member districts, rather via party list proportional representation, so I don't know that any elected representative has a specific single constituency they represent. --Jayron32 19:06, 2 October 2012 (UTC)Antwort
That is correct, members of the States-General do not represent any region or district any more than any other. They each represent all people that voted for their party, wherever they may live. However, Bonaire, Sint Eustatius and Saba don't have enough voters to significantly influence the outcome of the elections. Together they have about 13000 people that can vote, while the mainland has about 12.7 million, which is about 0.1%, i.e. 0.15 out of 150 seats in the Dutch House of Representatives. The Dutch senate is chosen indirectly, i.e. they are elected by the (directly elected) representatives of the 12 provinces and by the respective Island Councils of Bonaire, Sint Eustatius and Saba. The weight of the votes is proportional to the number of inhabitants they represent. - Lindert (talk) 19:32, 2 October 2012 (UTC)Reply

UN conventions and Denmark, Greenland and the Faroe Islands

If Denmark signs and ratifies United Nations conventions do they automatically include Greenland and the Faroes or do these two "autonomous countries under the Danish Crown" have the responsibility/competence to sign separately? The specific instance I'm concerned with is the Convention on the Rights of Persons with Disabilities but answering the general principle would be useful too. Roger (talk) 19:16, 2 October 2012 (UTC)Reply

There are examples when conventions do not effect Faroe or Greenland when they are for Denmark (the EU treaties are examples), but I have the feeling that in the case of Denmark the situation is: if the UN don't mention it, its Faroe, Greenland and Denmark. I checked the Dutch treaty database (not much of an authority in this case, but in my experience quite reliable); and they include Faroe and Greenland (see here)... L.tak (talk) 19:35, 2 October 2012 (UTC)Antwort
And the authoritive source! NOrmally (so: unless mentioned otherwise), they do include Faroe and Greenland. See the historical information section of the UN Treaty base. L.tak (talk) 19:41, 2 October 2012 (UTC)Reply
As to the general principle, neither Greenland nor the Faroes appears in the list of Member states of the United Nations, nor on the list of sovereign states. Also, there is this document submitted by Denmark on behalf of Denmark and Greenland [VERY LARGE PDF] detailing Danish actions under the Convention on the Law of the Sea relating to Greenland. So barring very special circumstances, the details of which would fall under WP:CRYSTAL, the Danish signature is the one that commits Greenland and the Faroes to participation. ☯.ZenSwashbuckler.☠ 19:41, 2 October 2012 (UTC)Antwort
I am a little confused. What does the Dutch treaty database have to do with Denmark? μηδείς (talk) 20:03, 2 October 2012 (UTC)Antwort
Let's mark call it resolved; we have the -very large pdf- of the Danish government and the short summary of the of UN Treaty series. (The Dutch treaty data base says the same. That is; as I said, not an authoritive source; but they simply record every treaty of interest to the Netherlands and they are good at it; that's why I sometimes go there in case of doubt; only later I found the more authoritive UN-source, that;s clearly a better one...). Rgds! L.tak (talk) 21:05, 2 October 2012 (UTC)Antwort
Interesting! Kind of like NOAA being responsible for the Boxing Day tsunami. μηδείς (talk) 21:52, 2 October 2012 (UTC)Reply
  Resolved

Thanks everyone. Roger (talk) 22:55, 2 October 2012 (UTC)Reply

Bruce Coville Story

For the class that I'm teaching I want to have the students read a short story by Bruce Coville about people that live in a giants teeth but I can't find it anywhere on the internet or libary. Does anyone know what the story is called and where I can buy a copy of it? Thanks. — Preceding unsigned comment added by 67.42.60.175 (talk) 19:20, 2 October 2012 (UTC)Reply

Search "giant's teeth bruce coville" at amazon. μηδείς (talk) 20:01, 2 October 2012 (UTC)Reply

Last Will and Testament

What percentage of the USA population dies intestate? — Preceding unsigned comment added by 68.205.201.84 (talk) 20:00, 2 October 2012 (UTC)Reply

Here and here and here are some starts. I found these using Google and Google scholar using the phrase "Americans die intestate". Its a start for you. --Jayron32 20:06, 2 October 2012 (UTC)Antwort

Death

Why do living organisms die? Is it possible for living organisms to live forever? I do recall there was a tree that lived for hundreds of years before it died. If a living organism has or is given an infinite amount of nutrients, then can it live forever? I know it sounds like science fiction, but can one defeat death if one were to remove all natural and unnatural causes of death? Or is death really inevitable and that there will always be some sort of unknown cause of death? Cell can't function anymore? Wear-and-tear? How do philosophers justify or explain death? 140.254.226.217 (talk) 20:16, 2 October 2012 (UTC)Reply

Living organisms die because they get eaten, injured, frozen, attacked by their own species, etc. They also run out of food or water, get diseases, suffer organ failures, etc. For organisms that don't have biological immortality, senescence kills cells because the mechanism for DNA replication fails to copy a short segment every time, and when this process starts cutting off useful DNA instead of just telomeres, cellular mechanisms start failing. The primary benefit of this seems to be in preventing cancer, and it almost always works; cancer cells can only proliferate because some mutation disables the mechanism and makes themselves immortal. Nothing can live forever because the stars will eventually die due to the second law of thermodynamics, and the universe will suffer a heat death (which is actually the coldest possible death). Finally, whatever philosophers think about death is utter bullshit unless they have empirical evidence for their claims, at which point they'd be scientists. --140.180.242.9 (talk) 20:50, 2 October 2012 (UTC)Reply
Oh. I'll take it as "There is no purpose in life" or "The meaning of life is to create a harmonious cycle of life or to behave naturally. Whatever that is supposed to mean is up to the individual." 140.254.226.217 (talk) 21:08, 2 October 2012 (UTC)Reply
I've unreddened and emblued your link (biological immorality), but its smirk-worthiness was lost in the process, damn it. Swings and roundabouts. -- Jack of Oz [Talk] 20:59, 2 October 2012 (UTC)Antwort
"Biological immorality" was pretty funny, but I'd rather 140.180 pay more attention to fixing his last sentence, which is wholly unproductive. While we don't have a "philosophy of death" article as such, death and culture is a reasonable starting point. Mortality salience and existential psychology may also be of interest. Naturally, no single answer can be given to "what do philosophers think of death?"; there are innumerably many philosophical approaches. — Lomn 21:03, 2 October 2012 (UTC)Reply
Naturally, death scares the hell out of me! 140.254.226.217 (talk) 21:11, 2 October 2012 (UTC)Reply
Note that many organisms could be "designed" to live much longer than they do, but short life spans are favored by evolution, as it allows for more generations, and hence more variation in a species, which permits it to adapt more quickly and effectively to it's constantly changing environment. Some organisms, like certain salmon, are programmed to die right after they breed. StuRat (talk) 21:16, 2 October 2012 (UTC)Antwort
That makes me feel better. Well, at least life has a purpose: to reproduce and help the population survive. Death, I suppose, is for a good cause. 140.254.226.217 (talk) 21:20, 2 October 2012 (UTC)Antwort
That's an odd perspective on the whole thing. You're dead FAR longer than you'll ever be alive. Life is a blink of the eye, a drop in the ocean. That's not to say it's irrelevant or trivial or unimportant, though. The question is not whether death has a good cause, but whether life can be made meaningful and fulfilling. I can guarantee that death will come to you; but I cannot guarantee you'll have a meaningful and fulfilling life in the meantime. That's up to you. -- Jack of Oz [Talk] 21:53, 2 October 2012 (UTC)Antwort
Although it's sad that good people have to die, it's balanced by the fact that bad people have to die also. If no one died, there would be no need to reproduce, nor any need to pass the torch to a new generation, as the old people would be around forever. Ugh. ←Baseball Bugs What's up, Doc? carrots02:01, 3 October 2012 (UTC)Reply
We inevitably die because we have sex. The two are even more closely linked than most people suppose.
Bacteria reproduce asexually by splitting. Each resulting bacterium has just as good a claim to be the original as the other one does, and every bacterium can be said to be as old as its species. With sex, though, that's all out the door. Two parents contribute their genes to make a new individual. Since the parents do not live on as their own descendants, the way bacteria do, they must inevitably die.
And the sad part is, that as a member of a sexually reproducing species you are doomed to die, whether or not you personally have sex. John M Baker (talk) 22:31, 2 October 2012 (UTC)Reply

Kant and Nietzsche's views

I cannot understand Nietzsche and Kant. It may be because of differences in manners of speaking arising through time. What were there views regarding what is right and what is wrong? Nietzsche is more interesting; of all the things I've read about him it just seems like he was a man who just put ideas and thoughts out in the open. Dd he actually believe any of the stuff he wrote? --Melab±1 23:38, 2 October 2012 (UTC)Reply

Both Nietzsche and Kant are quite difficult to read in the original, and they are written not only in a different style, but you're reading them in translation and in a world that isn't necessarily asking the same questions they were asking at the time. There isn't a way to summarize Nietzsche and Kant's views on "right and wrong" (or even "truth and falseness"!) in a short paragraph that wouldn't do great injustice to the nuance of their thinking. If you are looking for a nice way to dive into their thinking — or at least give you enough understanding of what their big questions are — I've found the A Very Short Introduction series to generally be pretty good. They have Kant and Nietzsche editions. --Mr.98 (talk) 01:20, 3 October 2012 (UTC)Reply
I strongly recommend Walter Kaufmann's translation of The Antichrist It is not at all hard to follow if you have the wider grounding that is assumed, like a familiarity with Judaism, Christianity, Ancient Rome, and Buddhism. Nietzsches' basic viewpoint is that moral systems are tools meant to achieve ends: validation of power and success (master morality) or validation of guilt and suffering (slave morality). You can read this work in two sittings, it is well worth the effort. μηδείς (talk) 01:37, 3 October 2012 (UTC)Reply
You may read Schopenhauer, who was the cerebral middleman between Kant and Nietzsche. The latter was strongly under his influence. --Omidinist (talk) 12:16, 3 October 2012 (UTC)Reply

October 3

Mombasa History

I'm currently working on the Mombasa article, and have not been able to find good history and culture resources by which to find and verify information. Does anybody have some recommendations with regard to sources? I'd also appreciate some "keywords" which I could use to search online, and get started. Thanks everybody! Van Gulik (talk) 00:39, 3 October 2012 (UTC)Reply

Muslim nations mizrahi and sephardi jews

Which Muslim nations formerly had Jewish population that they are called as Sephardi Jews in Israel? Which Muslim nations formerly had Jewish population that they are called as Mizrahi Jews in Israel?--70.29.32.229 (talk) 01:41, 3 October 2012 (UTC)Don MustafaReply

See Mizrahi Jews for the answer. Specifically, the opening paragraph (bold is mine):

Mizrahi Jews or Mizrahim (Hebrew: מזרחים‎), also referred to as Adot HaMizrach (עֲדוֹת-הַמִּזְרָח) (Communities of the East; Mizrahi Hebrew: ʿAdot(h) Ha(m)Mizraḥ), are Jews descended from the Jewish communities of the Middle East, North Africa and the Caucasus. The term Mizrahi is used in Israel in the language of politics, media and some social scientists for Jews from mostly Arab-ruled geographies and adjacent, primarily Muslim-majority countries. This includes Jews from Iraq, Syria, Bahrain, Saudi Arabia, Oman, United Arab Emirates, Jordan, Palestine, Lebanon, Yemen, Azerbaijan, Iran/Persia, Afghanistan, India, Uzbekistan, Kurdish areas, Northern and Eastern Sudan, as well as Ethiopia, and within and nearby Israel. Sometimes, Sephardi Jews such as Jews from Morocco, Algeria, Egypt, Tunisia, Libya or Turkey are erroneously grouped into the Mizrahi category for some historical reasons.

Hope it helps. --Jethro B 01:47, 3 October 2012 (UTC)Antwort
Sometimes Jews descended from ca. 1492 emigrants from Spain and/or who spoke Ladino are described as "Sephardi", while Jews who are neither Central-European influenced (Ashkenazi) nor Spanish influenced are called "Mizrachi". However, sometimes the distinction between Sephardi and Mizrachi is ignored. Yemen had a large Jewish community without any meaningful Spanish influences... AnonMoos (talk) 08:37, 3 October 2012 (UTC)Reply

Jews in Bangladesh

Which city of Bangladesh have historically had Jewish population? Someone told me it was Rajshahi but I didn't believe it. --70.29.32.229 (talk) 01:47, 3 October 2012 (UTC)Don Mustafa — Preceding unsigned comment added by 70.29.32.229 (talk) 01:46, 3 October 2012 (UTC)Reply

Typing "Jewish people in Bangladesh" into Google (you should try this sometime. Type exactly the question you want answered there first. It's a wealth of good information) turns up this article which states that there are between 175-3500 Jewish people in Bangladesh, depending on who you ask. At either end of that spectrum, that's a fairly tiny number. The article notes that many Jewish people in Bangladesh disguise their faith as they fear social retribution, so teasing out where most live may be difficult to do. --Jayron32 03:14, 3 October 2012 (UTC)Antwort
A good answer, but I believe the OP was referring to a few decades ago, when Bangladesh had a larger Jewish population. I could be wrong, but that's what I got from the OP writing "historically." I don't know the answer to that though. --Jethro B 03:32, 3 October 2012 (UTC)Antwort
Ah. In which case, a google search on History of judaism in bangladesh turns up some interesting stuff. Just this year, a former Jewish-Bangladeshi military officer was honored as a liberator of the country: [10]. Another source which looks promising to answer the OP's historical question is this one. --Jayron32 03:39, 3 October 2012 (UTC)Antwort

What Wikipedia has seems to be at Baghdadi Jews... AnonMoos (talk) 08:46, 3 October 2012 (UTC)Reply

Baghdad isn't in Bangladesh, though! Bangladeshi Jews is a redlink. --Tango (talk) 11:59, 3 October 2012 (UTC)Antwort
If you had looked at the article, "Baghdadi Jews" is a term for Jews who migrated from Baghdad at some point, apparently including a substantial fraction of former Bangladeshi Jews. In India, Baghdadi Jews are contrasted with Cochin Jews etc. AnonMoos (talk) 13:44, 3 October 2012 (UTC)Reply

Republic of Georgia building

What is the building at 3:09 of this video? 69.62.243.48 (talk) 03:11, 3 October 2012 (UTC)Reply

The maker of the video wrote in the comments, "That is from Batumi city at the Black Sea shore." --Jethro B 03:33, 3 October 2012 (UTC)Antwort
Yeah, but they didn't know what the building was. 69.62.243.48 (talk) 03:55, 3 October 2012 (UTC)Reply
Is this the building you mean: [11] ? Unfortunately, the only caption I see is "Divlek C. at Batumi City, Added June 19, 2012", which I take to be the name of the photographer. Here's the site where I found it: [12]. You might want to contact the blogger. StuRat (talk) 11:27, 3 October 2012 (UTC)Antwort
It's the Batumi Justice House. They are apparently dancing on the roof.--Cam (talk) 11:46, 3 October 2012 (UTC)Reply

Notorious lawyer known for defending war criminals, drug lords, terrorists, etc..

Not Giovanni Di Stefano or Jacques Vergès. It's Ludwig, Ludger something (I'm pretty confident on the "L") and he has a von or van attached to his name, but it was added for cosmetic reasons. He might have died recently. Eisenikov (talk) —Preceding undated comment added 05:15, 3 October 2012 (UTC)Reply

Edward von Kloberg III Found it by searching for "added "von" to his name" surprisingly it's a common practice. Eisenikov (talk) 05:31, 3 October 2012 (UTC)Reply

Are you sure "notorious" is really the right description? Everyone, even suspected drug bosses and terrorists have a right to be defended properly. Alansplodge (talk) 12:02, 3 October 2012 (UTC)Reply

Red, yellow, and green lights on the UN General Assembly podium

On the UN General Assembly podium there is a set of red, yellow, and green lights[13]. Is there an official statement from UN indicating its purpose? Most commentators assume it's a speech timer, but there are many flaws in that theory:

  1. Contrary to the commonly used green, yellow, red, flashing red sequence[14], the UN one goes from green to flashing red[15] directly. The yellow blub is never used as far as I can tell.
  2. In the above video the speaker goes on for 18 more minutes after the red light starts flashing. He essentially used more than double of his allotted time.
  3. The light is not used at all for some speakers[16].
  4. Speech timers are generally not visible to the audience, since it can be distracting. (Hence the question. During a boring speech all I can focus on is the blinking light.)

I'm looking for official UN documents describing the purpose of these indicator lights, or barring that, an authoritative source that describe it as a speech timer.A8875 (talk) 07:56, 3 October 2012 (UTC)Reply

I found some references to the light system (the link goes to one example of a search, I tried others too with other results). Sometimes the "orange" light is referred to and sometimes not. The time limits seem to vary by event.--Cam (talk) 12:12, 3 October 2012 (UTC)Reply

Dates of LaSalle's 1st (Ohio River) expedition

La Salle Expeditions I have done extensive searching for dates or even "three weeks later" and such of any of his 1669 voyage south of Western New York (specifically on the Allegheny River and upper Ohio River. I have searched all the links and references on the wiki articles for any specific dates south of western New York and even found this but alas no dates or time references. Anyone have a document or journal link to the complete journey? Marketdiamond (talk) 08:39, 3 October 2012 (UTC)Reply

Have you seen this? Starting at "In what regions" the author discusses the problem, including chronology issues and sources. He seems to conclude that there is no reliable evidence that La Salle explored the Ohio at that time. --Cam (talk) 12:35, 3 October 2012 (UTC)Reply

US presidential TV-debates

Good day all together. I am very interested in watching the TV-debates between President Barack Obama and Mitt Romney. I would like to watch them live but that's impossible because I live in Germany. So the debates take place in the middle of the night, therefore I have to watch it the next day. Does anyone know where I can watch them immedialtely? I doubt that someone will have uploaded it at youtube the same say. Perhaps CNN or CBS? Thanks in advance for the answers. --Jerchel (talk) 08:48, 3 October 2012 (UTC)Reply

C-SPAN will live stream it here[17].A8875 (talk) 08:53, 3 October 2012 (UTC)Reply
C-span will probably have videos online afterwards. There will also be transcripts, which are much faster to read than actually watching the videos. It might be worth watching a minute or two of video to get a look at the candidates, but watching the whole thing is in my experience a big waste of time. You get the info much faster from the transcripts. 67.117.130.72 (talk) 10:06, 3 October 2012 (UTC)Reply

Paramilitary education institutes in US

Hello guys! I tried to search here and on other wikis to find a list of paramilitary education institutes in US but I could not find. Could you give me a list? - Prücsök (talk) 09:20, 3 October 2012 (UTC)Reply

The really depends on how you define "paramilitary education institutes". If you're looking for places that will teach you how to shoot a gun, most ranges will offer that. If you're looking for PMC type companies that will offer the whole training package, then take a look at Category:Security_consulting_firms and Category:Private_military_contractors.A8875 (talk) 09:29, 3 October 2012 (UTC)Reply

Do you mean something like a List of United States military schools and academies? 67.117.130.72 (talk) 10:11, 3 October 2012 (UTC)Reply

This is what I though to. Thank you so much! - Prücsök (talk) 10:41, 3 October 2012 (UTC)Reply

Maria Francisca of Portugal

Infanta Maria Francisca of Portugal's article says "A crowd of several thousand were known to have visited the village to pay their respects as her body lay at the rectory awaiting burial at the Royal Pantheon of the Braganza Dynasty...Although initially interred in Gosport Catholic Church, Maria Francisca's remains were later transferred to Trieste Cathedral in Italy, next to those of her husband and children." Where exactly was she buried? How could she be transferred from the prestigious Braganza tombs to a church in Gosport, England, then to a Church in Italy? Was she ever buried in the Braganza tombs?--The Emperor's New Spy (talk) 13:36, 3 October 2012 (UTC)Reply

Which part of the paperclip product chain is, on average, the most profitable?

On average, when normalized to a per-paperclip figure, which has the highest profit margin, paperclip manufacturers who spend money on machines (amortized over life of machine, and maintenance), materials, and energy to actually make them and then sell them to wholesalers (or directly to retailers), paperclip wholesalers who spend money to buy them from manufacturers and then sell them to retailers, or retailers who spend money to get them and then sell them to customers? 20.137.18.53 (talk) 14:41, 3 October 2012 (UTC)Reply